OBGYN Flashcards

1
Q

A 32 yo F presents to her PCP complaining of painful periods, pain with sex, and painful bowel movements. She has been trying to have kids since she got married 2 years ago. A pelvic exam reveals a 6 cm tender mass in the rectouterine pouch (RUP) along with nodularity of the uterosacral ligaments.

A

This is endometriosis (deposition of endometrial tissue outside the uterus). There are many theories as to why this occurs but “retrograde menstruation” is a commonly peddled one. Consider this as your dx with mention of the 3D’s->dyschezia, dysmenorrhea, and dyspareunia. The most common location is the ovary (where it can bleed and cause an endometrioma) followed by the RUP. Definitive dx is with laparoscopy. Tx options include combined/progestin OCPs (nuke HPG axis), continuous GnRH (not pulsatile), and surgery if fertility is desired (or TAHBSO if postmenopausal and reproduction is no longer required).

How well did you know this?
1
Not at all
2
3
4
5
Perfectly
2
Q

Endometriosis is the most likely cause of ——— in a ——— woman over the age of ———, without a history of ———

A

infertility

menstruating

30

pelvic inflammatory disease.

How well did you know this?
1
Not at all
2
3
4
5
Perfectly
3
Q

A 37-year-old patient reports hemoptysis during her period.

A

Endometriosis of the nasopharynx or lung.

How well did you know this?
1
Not at all
2
3
4
5
Perfectly
4
Q

In endometriosis, severity of symptoms does not necessarily correlate with ———, but may correlate with the ———

A

quantity of ectopic endometrial tissue

depth of penetration of the ectopic tissue

How well did you know this?
1
Not at all
2
3
4
5
Perfectly
5
Q

Long-term complications of endometriosis:
Prolonged bleeding of ectopic tissue causes ———, may contribute to: (4)

A

scarring (adhesions).

infertility, small bowel obstruction, pelvic pain, and difficult surgeries.

How well did you know this?
1
Not at all
2
3
4
5
Perfectly
6
Q

Congenital anomalies that promote ——— may be found in adolescents with endometriosis.

A

retrograde menstruation

How well did you know this?
1
Not at all
2
3
4
5
Perfectly
7
Q

Chronic pelvic pain may result from endometriosis with associated

A

adhesive disease

How well did you know this?
1
Not at all
2
3
4
5
Perfectly
8
Q

Classic symptoms of endometriosis:

A

Dysmenorrhea, dyspareunia, and dyschezia

How well did you know this?
1
Not at all
2
3
4
5
Perfectly
9
Q

———- has been shown to reduce endometriosis-related dysmenorrhea.

A

Acupuncture

How well did you know this?
1
Not at all
2
3
4
5
Perfectly
10
Q

The pulsatile release of endogenous GnRH ——— FSH secretion. GnRH agonists cause ———- of pituitary receptors and ——— FSH secretion. This creates a ——— state.

A

stimulates

down regulation

suppress

pseudomenopause

How well did you know this?
1
Not at all
2
3
4
5
Perfectly
11
Q

The only way to definitively diagnose adenomyosis is with

A

microscopic examination of the uterus after hysterectomy.

How well did you know this?
1
Not at all
2
3
4
5
Perfectly
12
Q

When an enlarged uterus is found on exam, ultrasound can help differentiate between ——— and ———

A

adenomyosis and uterine fibroids.

How well did you know this?
1
Not at all
2
3
4
5
Perfectly
13
Q

The diagnosis of adenomyosis
is suggested by characteristic
clinical findings; ———-(3) after ——— and ——— have been ruled out)

A

heavy menses, dysmenorrhea, enlarged uterus

endometriosis

leiomyomas

How well did you know this?
1
Not at all
2
3
4
5
Perfectly
14
Q

Adenomyosis is classically described as an ——— uterus on physical exam.

A

enlarged, globular, “boggy”

How well did you know this?
1
Not at all
2
3
4
5
Perfectly
15
Q

Adenomyosis vs Endometriosis: Cyclic-ness of pain

A

Adenomyosis: Noncyclic pain
Endometriosis: Cyclic pain

How well did you know this?
1
Not at all
2
3
4
5
Perfectly
16
Q

Adenomyosis vs Endometriosis: Age and parity of women

A

Adenomyosis: Typically found in older, multiparous women
Endometriosis: Typically found in young, nulliparous women

How well did you know this?
1
Not at all
2
3
4
5
Perfectly
17
Q

Adenomyosis vs Endometriosis: Responsive to hormonal stimulation.

A

Adenomyosis: Tissue is not as responsive to hormonal stimulation

Endometriosis: Tissue is responsive to hormonal stimulation.

How well did you know this?
1
Not at all
2
3
4
5
Perfectly
18
Q

Oligomenorrhea is fewer than ——— menstrual cycles per year or cycle length of ——— days or more

A

nine

35

How well did you know this?
1
Not at all
2
3
4
5
Perfectly
19
Q

Primary amenorrhea:

A

Absence of menses by age 15 with normal growth and secondary sexual characteristics
or
absence of menses by age 13 with no secondary sexual characteristics

How well did you know this?
1
Not at all
2
3
4
5
Perfectly
20
Q

Secondary amenorrhea:

A

Absence of menses for ≥3 months in a woman who previously had a regular menstrual cycle
or
>6 months in a woman who had irregular menses

How well did you know this?
1
Not at all
2
3
4
5
Perfectly
21
Q

A 32-year-old G0P0 patient presents with a 3-year history of infertility. She
experienced menarche at age 13, and has regular menses every 28 days. She reports severe pain 2–3 days before her period, pain during her period, and pain with intercourse. She reports no history of sexually transmitted infections (STIs). Her husband has one child from a previous marriage. On exam, she has uterosacral nodularity and a fixed, retroflexed uterus. How should the suspected diagnosis be confirmed? What findings would be present on a tissue biopsy?

A

The patient has classic symptoms of endometriosis: dysmenorrhea and dyspareunia. Endometriosis is often associated with infertility. Although history and exam may suggest endometriosis, diagnostic laparoscopy is required to make a definitive diagnosis. The tissue biopsy would show endometrial glands, stroma, and hemosiderin-laden macrophages. The most common sites of involvement are the ovaries and pouch of Douglas.

How well did you know this?
1
Not at all
2
3
4
5
Perfectly
22
Q

A 39-year-old G4P4 patient presents with worsening heavy menstrual bleeding (HMB) and dysmenorrhea. On physical exam, the uterus is 14 weeks’ size, globular, boggy, slightly tender, and mobile. What is the next best step in management?

A

There is no proven medical therapy for adenomyosis, and hysterectomy is the only guaranteed treatment. However, conservative management with hormonal therapy is a reasonable next step. NSAIDs and OCPs can improve dysmenorrhea and regulate the heavy menses.

How well did you know this?
1
Not at all
2
3
4
5
Perfectly
23
Q

A 40 yo F complains of increasing pain with menses for the past 11 months. Her periods are extremely heavy. Her last child was delivered 12 years ago with a subsequent tubal ligation. On pelvic exam, her uterus appears enlarged and tender with a globular, soft consistency.

A

This patient has adenomyosis (the deposition of endometrial glands in the myometrium). In contrast with a leiomyoma that has an asymmetric, firm, and nontender uterine presentation, adenomyosis has a symmetric, soft, and tender uterine presentation. Dx is usually clinical although an MRI may be helpful. It is very HY to know that dx can only be made conclusively by the examination of tissue after surgery. Tx is usually with the levonorgestrel IUD which may curb the menstrual bleeding. Definitive treatment revolves around getting a hysterectomy.

How well did you know this?
1
Not at all
2
3
4
5
Perfectly
24
Q

When evaluating a patient with primary amenorrhea, note presence/absence of

A

breasts and uterus

How well did you know this?
1
Not at all
2
3
4
5
Perfectly
25
Q

Primary amenorrhea + elevated serum FSH =

A

Gonadal dysgenesis. Most common cause of primary amenorrhea ~40–45%.

How well did you know this?
1
Not at all
2
3
4
5
Perfectly
26
Q

17α hydroxylase deficiency:
46,XX: Breast ———, uterus ———; 46,XY: Breast ———, uterus ———

A

46,XX: Breast absent, uterus present 46,XY: Breast absent, uterus absent

How well did you know this?
1
Not at all
2
3
4
5
Perfectly
27
Q

Androgen insensitivity:

A

Patients look female externally. No pubic hair. Remove gonads after puberty to avoid risk of malignancy (gonadoblastoma or dysgerminoma).

How well did you know this?
1
Not at all
2
3
4
5
Perfectly
28
Q

An 18-year-old G0 patient presents with primary amenorrhea. Her sister
experienced menarche at age 12. She reports no use of drugs, heavy exercise, or significant weight loss. She is 5′5′′ and 130 lb. Her blood pressure is 110/60. Physical exam demonstrates Tanner stage IV breasts, no axillary or pubic hair, and a blind vaginal pouch. What is the most likely diagnosis?

A

Answer: Androgen insensitivity. Breasts are present; uterus and axillary/pubic hair is absent in androgen insensitivity.

How well did you know this?
1
Not at all
2
3
4
5
Perfectly
29
Q

What is the result of Müllerian agenesis?

A

Absent uterus

How well did you know this?
1
Not at all
2
3
4
5
Perfectly
30
Q

Second most common cause of primary amenorrhea

A

Müllerian agenesis

How well did you know this?
1
Not at all
2
3
4
5
Perfectly
31
Q

Normal breast and pubic hair + no menses + cyclic pelvic pain + bulging blue mass at the introitus =

A

Hematocolpos from imperforate hymen

How well did you know this?
1
Not at all
2
3
4
5
Perfectly
32
Q

Androgen Insensitivity vs Müllerian Agenesis: Karyotype

A

Androgen Insensitivity: XY

Müllerian Agenesis: XX

How well did you know this?
1
Not at all
2
3
4
5
Perfectly
33
Q

Androgen Insensitivity vs Müllerian Agenesis: Breast

A

Androgen Insensitivity: Present

Müllerian Agenesis: Present

How well did you know this?
1
Not at all
2
3
4
5
Perfectly
34
Q

Androgen Insensitivity vs Müllerian Agenesis: Uterus

A

Androgen Insensitivity: Absent

Müllerian Agenesis: Absent

How well did you know this?
1
Not at all
2
3
4
5
Perfectly
35
Q

Androgen Insensitivity vs Müllerian Agenesis: Pubic/axillary hair

A

Androgen Insensitivity: Absent

Müllerian Agenesis: Normal

How well did you know this?
1
Not at all
2
3
4
5
Perfectly
36
Q

Androgen Insensitivity vs Müllerian Agenesis: Testosterone

A

Androgen Insensitivity: Normal male levels

Müllerian Agenesis: Female levels

How well did you know this?
1
Not at all
2
3
4
5
Perfectly
37
Q

Androgen Insensitivity vs Müllerian Agenesis: Further evaluation

A

Androgen Insensitivity: Need gonadectomy

Müllerian Agenesis: Renal/skeletal abnormalities

How well did you know this?
1
Not at all
2
3
4
5
Perfectly
38
Q

The most common cause of secondary amenorrhea is

A

pregnancy. Always check a pregnancy test in a reproductive-age woman.

How well did you know this?
1
Not at all
2
3
4
5
Perfectly
39
Q

A 30-year-old G2P2002 patient, with last menstrual period (LMP) 8 weeks ago, presents with no menses for 2 months. She usually has menses every 28 days lasting for 5 days. She reports no medical or surgical history. She has had two vaginal deliveries at term. She uses combination oral contraceptive pills (OCPs), and has not missed any pills recently. What is the next step in management of this patient?

A

The most common cause of amenorrhea in a reproductive-age woman is pregnancy, so a urine or serum β-hCG should be checked. Contraception use does not prevent pregnancy 100% of the time.

How well did you know this?
1
Not at all
2
3
4
5
Perfectly
40
Q

Adolescent girls with amenorrhea should be screened for

A

disordered eating patterns

How well did you know this?
1
Not at all
2
3
4
5
Perfectly
41
Q

What is Sheehan syndrome?

A

Postpartum pituitary dysfunction due to intrapartum ischemia

How well did you know this?
1
Not at all
2
3
4
5
Perfectly
42
Q

In Sheehan syndrome, patients do not produce ——— due to ———

A

milk

the absence of prolactin

How well did you know this?
1
Not at all
2
3
4
5
Perfectly
43
Q

Sheehan syndrome typically presents with —(2)— after delivery in a patient with a history of ———

A

failure of postpartum lactation and failure to resume menses

severe postpartum hemorrhage

How well did you know this?
1
Not at all
2
3
4
5
Perfectly
44
Q

Sheehan syndrome: ——— occurs due to severe ——— resulting in hypotension and often requiring blood transfusion. Treatment includes ———

A

Pituitary cell infarction

post-partum hemorrhage

replacement of pituitary hormones

How well did you know this?
1
Not at all
2
3
4
5
Perfectly
45
Q

Primary ovarian insufficiency:
3 key elements presentation

A

■ Age <40
■ Amenorrhea
■ Elevated FSH

How well did you know this?
1
Not at all
2
3
4
5
Perfectly
46
Q

A 35-year-old G2P2002 patient with LMP 1 year ago presents with hot flashes and vaginal dryness. Her serum FSH is very high. What is the most likely diagnosis?

A

Primary ovarian insufficiency. Symptoms are similar to those in menopause and diagnosis is confirmed with elevated FSH

How well did you know this?
1
Not at all
2
3
4
5
Perfectly
47
Q

Patients who exercise or play sports intensely may develop the “female athlete triad,” characterized by

A

amenorrhea, disordered eating, and decreased bone mineral density. The primary issue is energy imbalance, whereby more calories are expended than consumed.

How well did you know this?
1
Not at all
2
3
4
5
Perfectly
48
Q

——— is the most common cause of hirsutism.

A

PCOS

How well did you know this?
1
Not at all
2
3
4
5
Perfectly
49
Q

A 35-year-old G3P3003 patient presents with absence of menses for 8 months. She reports menarche at age 12 with menses every 40–50 days until recently. She reports a 20-lb weight gain over the last year. She used letrozole to become pregnant with her last two pregnancies. Vitals show height 5′4′′, weight 220 lb, BP 120/80. She has hair on her upper lip and chin, acne, and oily skin on her face. What is the most likely diagnosis? If left untreated, what is this patient at ↑ risk for?

A

Polycystic ovarian syndrome (PCOS). Diagnosis of PCOS is established with two out of three of the following: a history of oligomenorrhea/amenorrhea, features of hyperandrogenism (acne, hirsutism), and multiple ovarian cysts seen on ultrasound. This patient is at ↑ risk for endometrial hyperplasia or cancer if left untreated.

How well did you know this?
1
Not at all
2
3
4
5
Perfectly
50
Q

A 28-year-old G0 patient has been unable to conceive with her husband over 1 year of regular, unprotected intercourse. Her periods are irregular. She has a BMI of 30, displays coarse facial hair, and a dark velvety pigmentation on the back of her neck. What is the likely diagnosis in this patient? What is the most likely reason she is unable to conceive?

A

Polycystic ovarian syndrome (PCOS) affects approximately 5% of all women, and is a leading cause of infertility. Her exam shows evidence of hyperandrogenism and insulin resistance. She is anovulatory and will likely need an ovulation induction agent to conceive.

How well did you know this?
1
Not at all
2
3
4
5
Perfectly
51
Q

Treatment of choice for PCOS:

A

Combined OCPs as they manage hyperandrogenism, menstrual irregularities, and provide contraception.

How well did you know this?
1
Not at all
2
3
4
5
Perfectly
52
Q

Asherman syndrome: Intrauterine adhesions can

A

obliterate the endometrial cavity and cause amenorrhea.

How well did you know this?
1
Not at all
2
3
4
5
Perfectly
53
Q

Asherman syndrome is ——— secondary to ———

A

intrauterine adhesions

uterine curettage associated with pregnancy (including after SAB, for retained placenta, or PPH) or shortly thereafter with resultant scarring.

How well did you know this?
1
Not at all
2
3
4
5
Perfectly
54
Q

The most common symptoms associated with Asherman syndrome include (4)

A

amenorrhea or light periods, infertility, cyclic pelvic pain, recurrent pregnancy loss.

How well did you know this?
1
Not at all
2
3
4
5
Perfectly
55
Q

Progestin challenge test:

A

Give oral progestin for 10 days. If the endometrium has been primed with estrogen from ovaries or peripheral fat, the withdrawal of progestin after 10 days will cause endometrial sloughing with resultant withdrawal bleed. No withdrawal bleeding indicates absence of ovaries, estrogen deficiency, or outflow obstruction.

How well did you know this?
1
Not at all
2
3
4
5
Perfectly
56
Q

Consider Asherman syndrome as the dx if a woman has ——— and the Q stem gives you a history of (2)

A

amenorrhea

dilation and curettage or super bad PID

(In some cases, these “uterine disturbances” create scar tissue that can cause amenorrhea/infertility)

How well did you know this?
1
Not at all
2
3
4
5
Perfectly
57
Q

Asherman syndrome treatment

A

Hysteroscopic lysis of these adhesions + estrogen typically is a good management step.

How well did you know this?
1
Not at all
2
3
4
5
Perfectly
58
Q

Do not forget to order a ——— in a woman presenting with amenorrhea

A

serum B-HCG

(In addition, consider taking a look at the TSH, prolactin levels (give Cabergoline as tx), androgen levels (testosterone and DHEAS), etc. )

How well did you know this?
1
Not at all
2
3
4
5
Perfectly
59
Q

A woman with PCOS will have a ———progesterone withdrawal challenge

A

+ve

How well did you know this?
1
Not at all
2
3
4
5
Perfectly
60
Q

A woman with Asherman’s syndrome/some kind of obstructive lesion in the uterus/cervix will have a ———progesterone withdrawal challenge.

A

-ve

(This will also be the case with someone having low E2 levels.)

How well did you know this?
1
Not at all
2
3
4
5
Perfectly
61
Q

The Tanner stages of development refer to the ———; these are———

A

sequence of events of breast and pubic hair development.

Stage 1: Prepuberty
Stages 2–4: Development stages
Stage 5: Adult development

How well did you know this?
1
Not at all
2
3
4
5
Perfectly
62
Q

The first menstrual bleeding is usually caused by the effects of——— stimulating the ———, rather than by ovulation.

A

estradiol

endometrial lining

How well did you know this?
1
Not at all
2
3
4
5
Perfectly
63
Q

Puberty is believed to begin with ———

A

disinhibition of the pulsatile GnRH secretion from the hypothalamus

How well did you know this?
1
Not at all
2
3
4
5
Perfectly
64
Q

GnRH stimulates the

A

anterior pituitary gland to secrete follicle-stimulating hormone (FSH) and luteinizing hormone (LH)

How well did you know this?
1
Not at all
2
3
4
5
Perfectly
65
Q

At puberty in girls, FSH stimulates ———, and along with LH, stimulates production of ——— (which causes (3))

A

growth of ovarian follicles

estradiol in the ovaries

breast development, skeletal growth, and stimulation of the endometrium

(Later, FSH and LH lead to ovulation and menstruation)

How well did you know this?
1
Not at all
2
3
4
5
Perfectly
66
Q

Thelarche is ———; average age ———; dominant hormone ———

A

breast budding

10

estradiol

How well did you know this?
1
Not at all
2
3
4
5
Perfectly
67
Q

Pubarche is ———; average age ———; dominant hormone ———

A

axillary and pubic hair growth

11

Adrenal hormones (androgens)

How well did you know this?
1
Not at all
2
3
4
5
Perfectly
68
Q

Menarche is ———; average age ———; dominant hormone ———

A

first menses

12

Estradiol

How well did you know this?
1
Not at all
2
3
4
5
Perfectly
69
Q

A female age ——— or older without any breast development should be evaluated for delayed puberty.

A

13

How well did you know this?
1
Not at all
2
3
4
5
Perfectly
70
Q

Menstrual cycle is from he HPO axis and uterus interacting to allow ovulation approximately ——— [average ——— days (+/– ———days)].

A

once per month

28

7

How well did you know this?
1
Not at all
2
3
4
5
Perfectly
71
Q

In menstruation, many follicles are stimulated by ———, but the follicle that ——— will be released

A

FSH

secretes more estrogen than androgen

How well did you know this?
1
Not at all
2
3
4
5
Perfectly
72
Q

In menstruation, the dominant follicle ——— so that its positive feedback causes an ———

A

releases the most estradiol

LH surge

How well did you know this?
1
Not at all
2
3
4
5
Perfectly
73
Q

Menstruation: Withdrawal of ——— causes ———

A

progesterone

endometrial sloughing

How well did you know this?
1
Not at all
2
3
4
5
Perfectly
74
Q

■ Average duration of menses =
■ Normal menstrual cycle length =

A

5 days

21–35 days

How well did you know this?
1
Not at all
2
3
4
5
Perfectly
75
Q

——— released from the ——— cause dysmenorrhea.

A

Prostaglandins

endometrium

How well did you know this?
1
Not at all
2
3
4
5
Perfectly
76
Q

Before ovulation vs after ovulation: ovarian phase

A

Before ovulation: follicular

After ovulation: luteal

How well did you know this?
1
Not at all
2
3
4
5
Perfectly
77
Q

Before ovulation vs after ovulation: uterine phase

A

Before ovulation: proliferative

After ovulation: secretory

How well did you know this?
1
Not at all
2
3
4
5
Perfectly
78
Q

Before ovulation vs after ovulation: dominant hormone

A

Before ovulation: estrogen

After ovulation: progesterone

How well did you know this?
1
Not at all
2
3
4
5
Perfectly
79
Q

Follicular phase: FSH causes (2)

A

follicle maturation and estrogen secretion

How well did you know this?
1
Not at all
2
3
4
5
Perfectly
80
Q

Follicular phase (day 1-14): Estrogen causes

A

endometrial proliferation

How well did you know this?
1
Not at all
2
3
4
5
Perfectly
81
Q

Ovulation (Day 14): The LH surge causes the ———. The ruptured follicle then becomes the ———, which secretes ———.

A

oocyte to be released from the follicle

corpus luteum

progesterone

How well did you know this?
1
Not at all
2
3
4
5
Perfectly
82
Q

Luteal phase (14-28): Corpus luteum secretes ———, which causes (2)

A

progesterone

Endometrial maturation and↓FSH,↓LH

How well did you know this?
1
Not at all
2
3
4
5
Perfectly
83
Q

Ovulation takes place ——— hours after LH surge and ——— hours after LH peak.

A

24–36

12

How well did you know this?
1
Not at all
2
3
4
5
Perfectly
84
Q

The length of the ——— phase is highly variable. The ——— phase is usually about 14 days due to the ———. Individuals who have changes in their cycle length typically experience changes in the ——— phase.

A

follicular

luteal

length of time the corpus luteum is able to secrete progesterone

follicular

How well did you know this?
1
Not at all
2
3
4
5
Perfectly
85
Q

The corpus luteum is maintained after fertilization by ———, which is released by ———.

A

hCG

embryo

How well did you know this?
1
Not at all
2
3
4
5
Perfectly
86
Q

After progestin challenge test, give ——— to Distinguish Between Hypoestrogenism or an Outflow Tract Obstruction (Asherman’s Syndrome or Cervical Stenosis):
If bleeding occurs, amenorrhea is due to ———
If bleeding does not occur, then most likely it is ———

A

Estrogen and Progestin (Give estrogen to ensure endometrial proliferation, followed by a progestin to induce withdrawal bleeding. )

hypoestrogenism (hypothalamic amenorrhea or premature ovarian failure)

outflow tract obstruction – either Asherman’s syndrome or cervical stenosis.

How well did you know this?
1
Not at all
2
3
4
5
Perfectly
87
Q

“bluish bulge” in the vagina associated with ———; treated with ———

A

an imperforate hymen

“cruciate incision”

How well did you know this?
1
Not at all
2
3
4
5
Perfectly
88
Q

23 yo F has had menses before, missed her last 3 periods, urine B-HCG is +ve

A

she’s pregnant

How well did you know this?
1
Not at all
2
3
4
5
Perfectly
89
Q

23 yo F has had menses before, missed her last 3 periods, biopsy reveals lymphoid follicles in the thyroid

A

Hashimoto’s, high TRH, hyperprolactinemia

How well did you know this?
1
Not at all
2
3
4
5
Perfectly
90
Q

23 yo F has had menses before, missed her last 3 periods, has galactorrhea

A

prolactinoma (tx with bromocriptine, cabergoline)

How well did you know this?
1
Not at all
2
3
4
5
Perfectly
91
Q

23 yo F has had menses before, missed her last 3 periods, was diagnosed with schizophrenia 3 mo ago

A

dopamine antagonists, hyperprolactinemia.

How well did you know this?
1
Not at all
2
3
4
5
Perfectly
92
Q

35 yo F has had menses before, BMI of 32, irregular menses for the past 5 years

A

PCOS

How well did you know this?
1
Not at all
2
3
4
5
Perfectly
93
Q

Gestational age:

A

The time of pregnancy counting from the first day of the last menstrual period (LMP).

How well did you know this?
1
Not at all
2
3
4
5
Perfectly
94
Q

First trimester:

Second trimester:

Third trimester:

A

First trimester: 0–12 weeks.

Second trimester: 13–27 weeks.

Third trimester: 28 weeks–birth.

How well did you know this?
1
Not at all
2
3
4
5
Perfectly
95
Q

Embryo:

Fetus:

A

Embryo: Fertilization–8 weeks.

Fetus: 9 weeks–birth.

How well did you know this?
1
Not at all
2
3
4
5
Perfectly
96
Q

Previable:

Periviable:

Preterm:

Term:

A

Previable: <22 weeks.

Periviable: 22–24 weeks.

Preterm: 20–36 weeks.

Term: 37–42 weeks.

How well did you know this?
1
Not at all
2
3
4
5
Perfectly
97
Q

Remember that ——— has the highest correlation with congenital abnormalities.

A

alcohol use

How well did you know this?
1
Not at all
2
3
4
5
Perfectly
98
Q

20 yo F has breasts and a uterus, levels of all hormones are normal

A

imperforate hymen, or anything weird that blocks the vagina. She is actually menstruating, but the blood is not making its way out. Dx is by PE, tx is with surgery.

How well did you know this?
1
Not at all
2
3
4
5
Perfectly
99
Q

20 yo F supermodel (or hardcore athlete) has breasts and a uterus (what should be true of her hormone levels?)

A

this is either anorexia nervosa/just working out too much. In the setting of severe physiologic stress, the GnRH axis is turned off. Therefore, GnRH/LH/FSH/Estrogen levels are all low.

How well did you know this?
1
Not at all
2
3
4
5
Perfectly
100
Q

20 yo F has breasts and no uterus, testosterone levels are super high

A

she has no uterus b/c she is 46 XY. this is testicular feminization syndrome (or Androgen Insensitivity Syndrome)- Sertoli cells make MIH so the mullerian duct is nuked (oviducts, uterus, upper third of the vagina). She has breasts b/c the testosterone is aromatized to estrogen in the periphery. Why should she have an orchiectomy after puberty?

How well did you know this?
1
Not at all
2
3
4
5
Perfectly
101
Q

20 yo F has breasts and no uterus, karyotype reveals a 46 XX phenotype

A

Mullerian Agenesis (Mayer Rokitansky Kuster Hauser Syndrome). For whatever reason, the mullerian duct does not develop so there’s no uterus. She has ovaries which produce estrogen (ovaries are not derived from the mullerian ducts) so she has breasts. Whenever you see breasts, estrogens are around. Whenever you see pubic hair, androgens are around.

How well did you know this?
1
Not at all
2
3
4
5
Perfectly
102
Q

20 yo F has no breasts, but has a uterus, she can’t smell

A

Kallmann Syndrome, everything will be low-GnRH down. No breasts b/c the ovaries are making no estrogen. Give pulsatile leuprolide.

How well did you know this?
1
Not at all
2
3
4
5
Perfectly
103
Q

20 yo F is 3’ 5” tall, low posterior hairline, widely spaced nipples, BP of 150/120 in the arms and a BP of 65/40 in the legs, has a uterus, no breasts

A

this is Turner’s syndrome. Turner’s is a kind of hypergonadotropic hypogonadism (streak ovaries) so the FSH levels will be high. Karyotype is 45 XO. Common associations include bicuspid aortic valves, coarctation of the aorta (BP differences b/w the arms and legs), horseshoe kidney, amongst other anomalies. With the ovaries basically non-existent, there is no estrogen (or very little), so there is no breast development.

How well did you know this?
1
Not at all
2
3
4
5
Perfectly
104
Q

20 yo F with a uterus, no breasts, and visual field deficits

A

some kind of brain tumor (like a craniopharyngioma, considering the VF deficits). A compressive lesion of the anterior pituitary may hamper LH/FSH production so the ovaries are not stimulated (so there’s no estrogen = no breasts).

How well did you know this?
1
Not at all
2
3
4
5
Perfectly
105
Q

A 31-year-old G2P1001 patient at 17 weeks’ gestation undergoes routine prenatal tests. Her results show that her blood type is A negative, and her antibody screen is positive. She does not report undergoing a blood transfusion in the past or any complications with her last pregnancy. What is the next step in the management of this patient?

A

The next step is to identify the antibody. There are many types of antibodies, and in a patient that is Rh negative, it should not be assumed that she has Rh antibodies.

How well did you know this?
1
Not at all
2
3
4
5
Perfectly
106
Q

The only contraceptive methods that protect against STIs are

A

the male and female condoms

How well did you know this?
1
Not at all
2
3
4
5
Perfectly
107
Q

——— accounts for most condom failures.

A

Inconsistent condom use

How well did you know this?
1
Not at all
2
3
4
5
Perfectly
108
Q

If diaphragm left in for too long (>24 hours), in rare cases, may result in

A

Staphylococcus aureus infection (which may cause toxic shock syndrome).

How well did you know this?
1
Not at all
2
3
4
5
Perfectly
109
Q

Efficacy rates for spermicides are much higher when

A

combined with other barriers (i.e., condoms, diaphragms).

How well did you know this?
1
Not at all
2
3
4
5
Perfectly
110
Q

A 37-year-old G2P2 patient desires a reversible form of contraception. Her history is significant for migraines with visual aura, uncontrolled hypertension (HTN), and family history of breast cancer in her mother. She smokes two packs of cigarettes daily. She requests combination oral contraceptives (COCs). How should this patient be counseled?

A

The patient is not a candidate for COCs because she has several contra- indications that put her at an increased risk for developing venous thromboembolism and stroke. Contraindications for COC use include age >35 years old and smoking, history of venous thromboembolism, uncontrolled HTN, diabetes with vascular disease, migraines with visual aura, and benign or malignant liver tumors, cirrhosis, and personal history of breast cancer.

How well did you know this?
1
Not at all
2
3
4
5
Perfectly
111
Q

——— headaches are not a contraindication for COCs. ——— headaches can increase risk of stroke in patients who take combination hormonal contraception.

A

Tension

Migraine with visual aura

How well did you know this?
1
Not at all
2
3
4
5
Perfectly
112
Q

P450 inducers will decrease the efficacy of ——— contraceptives (e.g., phenytoin, rifampin, griseofulvin, carbamazepine, barbiturates) due to accelerated metabolism of ———.

A

oral

OCPs

How well did you know this?
1
Not at all
2
3
4
5
Perfectly
113
Q

Hormonal patch may be less effective in

A

obese (≥200 lb) women.

How well did you know this?
1
Not at all
2
3
4
5
Perfectly
114
Q

Types of endogenous estrogens:
Reproductive life: ———
Pregnancy: ———
Menopause: ———

A

Estradiol

Estriol

Estrone

(Estrone (E1): People make this type of estrogen after menopause.
Estradiol (E2): This is the most common type of estrogen that people make.
Estriol (E3): This is the main type of estrogen in a pregnant person’s bloodstream.)

How well did you know this?
1
Not at all
2
3
4
5
Perfectly
115
Q

The inactive pills in the COC simulate ———, which results in menses.

A

hormone withdrawal of the normal menstrual cycle

How well did you know this?
1
Not at all
2
3
4
5
Perfectly
116
Q

Always check a β-hCG to rule out pregnancy before prescribing the acne medicine ——— (very teratogenic!).

A

isotretinoin

How well did you know this?
1
Not at all
2
3
4
5
Perfectly
117
Q

What is the treatment for idiopathic hirsutism?

A

OCPs

How well did you know this?
1
Not at all
2
3
4
5
Perfectly
118
Q

Oral contraception mechanism in a nutshell:
■ Estrogen inhibits ———
■ Progestin inhibits ———

A

FSH (prevents follicular development)

LH (inhibits ovulation)

How well did you know this?
1
Not at all
2
3
4
5
Perfectly
119
Q

COC: Estrogen and progesterone combined. Main mechanisms: (4)

A

■ Prevents ovulation
■ Alters uterine and fallopian tube motility
■ Thickens cervical mucus to prevent
sperm penetration
■ Causes endometrial atrophy

How well did you know this?
1
Not at all
2
3
4
5
Perfectly
120
Q

There is no proven link between OCP use and ↑ in

A

breast cancer

How well did you know this?
1
Not at all
2
3
4
5
Perfectly
121
Q

Estrogen can impact ———, so combination pills are not recommended for ———.
——— are recommended until that time.

A

breast milk supply and production

nursing mothers until milk supply is stable

Progestin-only pills

How well did you know this?
1
Not at all
2
3
4
5
Perfectly
122
Q

Side effects of estrogen: (3)

A

■ Breast tenderness
■ Nausea
■ Headache

How well did you know this?
1
Not at all
2
3
4
5
Perfectly
123
Q

Side effects of progestin: (4)

A

■ Depression
■ Acne
■ Weight gain
■ Irregular bleeding

How well did you know this?
1
Not at all
2
3
4
5
Perfectly
124
Q

Why is estrogen a procoagulant?

A

Estrogen ↑ factors VII and X and ↓ antithrombin III.

How well did you know this?
1
Not at all
2
3
4
5
Perfectly
125
Q

A 20-year-old G0 patient desires long-acting reversible contraception. She has a history of epilepsy for which she takes an anticonvulsant. She still has seizures once about every 6 months. She is also wary of anything that goes ‘in her body’. What is the best contraceptive method for her?

A

Medroxyprogesterone acetate (DMPA) injection can ↑ the seizure threshold and ↓ the number of seizures. It also ↓ the number of sickle cell crises in patients with sickle cell disease. It improves anemia, ↓ dysmenorrhea and ovarian cysts, and improves symptoms of endometriosis.

How well did you know this?
1
Not at all
2
3
4
5
Perfectly
126
Q

Most common complaint / reason for stopping Medroxyprogesterone acetate (DMPA) injection:

A

Weight gain

How well did you know this?
1
Not at all
2
3
4
5
Perfectly
127
Q

Non-user-dependent methods like the IUD, subdermal implant, and injections have lower failure rates than ———.

A

OCPs

How well did you know this?
1
Not at all
2
3
4
5
Perfectly
128
Q

IUDs and contraceptive implants, also called long-acting reversible contraceptives (LARC), are the most effective

A

reversible contraceptive methods

How well did you know this?
1
Not at all
2
3
4
5
Perfectly
129
Q

If the IUD strings are not visible on follow-up speculum examination, a cytobrush can be used to ———. If this is not successful, then an ——— should be performed to determine the location of the IUD. If the IUD is not visible in the uterus on ———, then an ——— should be performed to assess for possible ———.

A

try to pull the strings down and out of the cervical os.

ultrasound

ultrasound

abdominopelvic X-ray

uterine perforation

How well did you know this?
1
Not at all
2
3
4
5
Perfectly
130
Q

A 25-year-old G1P1patient, who delivered a full-term infant 6 months previously,
presents for IUD insertion. She reports that she is in a long-term, monogamous relationship. She reports no history of STIs or other medical conditions. She undergoes the insertion of an IUD without any apparent complications. The patient presents 4 days later with abdominal pain, nausea, vomiting, and fever. Speculum exam reveals malodorous discharge and IUD strings at the cervical os. What is the most likely cause for the patient’s symptoms?

A

Endometritis due to contamination during insertion. Infections that occur near the time of IUD insertion are usually due to ascending infection from vaginal flora. Infections, months to years, after the IUD placement may be due to STIs.

How well did you know this?
1
Not at all
2
3
4
5
Perfectly
131
Q

Contraindication for IUD placement: ———, due to ———

A

Severe distortion of uterine cavity (e.g., bicornuate uterus or large obstructive fibroids)

increased difficulty with insertion, increased risk of IUD expulsion/ embedment/perforation, and decreased efficacy.

How well did you know this?
1
Not at all
2
3
4
5
Perfectly
132
Q

The absolute risk of ——— is lower for women using an IUD compared with women not using contraception or using other reversible methods of contraception. However, should a pregnancy occur in a patient using and IUD, the risk of ——— is higher, ranging from 10 to 30%.

A

ectopic pregnancy

ectopic pregnancy

How well did you know this?
1
Not at all
2
3
4
5
Perfectly
133
Q

——— are the most effective method of emergency contraception (both types are more than 99% effective). Another benefit of this method is that they may be ———

A

IUDs

left in place in order to continue to provide ongoing, highly effective contraception.

How well did you know this?
1
Not at all
2
3
4
5
Perfectly
134
Q

The only contraceptive methods that protect against STDs are

A

abstinence/condoms.

How well did you know this?
1
Not at all
2
3
4
5
Perfectly
135
Q

Combined OCPs work by (2)

A

inhibiting ovulation (E2 -ve feedback)

ncreasing the thickness of cervical mucus (progesterone)

How well did you know this?
1
Not at all
2
3
4
5
Perfectly
136
Q

Avoid combined OCPs in individuals with (7)

A

history of a DVT/PE

Weird genetic diseases like Factor 5 Leiden

history of cancers driven by E2 (like breast)

people with bad HTN

patients with hepatic adenomas

smokers > 35 yo

h/o migraine with aura/atypical migraine involving neuro deficits.

How well did you know this?
1
Not at all
2
3
4
5
Perfectly
137
Q

Combined OCPs decrease the risk of

A

ovarian/endometrial cancer (fewer cycles)

How well did you know this?
1
Not at all
2
3
4
5
Perfectly
138
Q

If a lady is being treated for TB and becomes pregnant with regular combined OCP use, think of

A

revved up metabolism from rifampin mediated CYP450 induction.

How well did you know this?
1
Not at all
2
3
4
5
Perfectly
139
Q

Progestin only pills MOA involves thickening

A

cervical mucus

How well did you know this?
1
Not at all
2
3
4
5
Perfectly
140
Q

Progestin only pills associated with (2)

A

weight gain

reversible osteoporosis.

How well did you know this?
1
Not at all
2
3
4
5
Perfectly
141
Q

Progestin only pills are good ——— options

A

postpartum

How well did you know this?
1
Not at all
2
3
4
5
Perfectly
142
Q

A progesterone derivative (Megestrol) is given to

A

stimulate the appetite in cachectic patients

How well did you know this?
1
Not at all
2
3
4
5
Perfectly
143
Q

The Cu IUD is an excellent contraceptive that works by causing

A

an inflammatory reaction that makes the uterus inhospitable for sperm and eggs

How well did you know this?
1
Not at all
2
3
4
5
Perfectly
144
Q

Remember the association of ———with a history of tubal ligation.

A

ectopic pregnancies

How well did you know this?
1
Not at all
2
3
4
5
Perfectly
145
Q

The Cu IUD is should be avoided in a woman with (2)

A

History of heavy menses

Wilson’s disease

How well did you know this?
1
Not at all
2
3
4
5
Perfectly
146
Q

A 22 yo G1P1 female visits her obstetrician 1 week after she delivered a boy that weighed 6 Lbs and 4 ounces. The delivery was unremarkable with APGAR scores of 8 and 9 at 1 and 5 minutes respectively. The patient plans to have her next baby when she graduates from graduate school 2 years from now. In addition to routine screening for postpartum depression, what is the next best step in the management of this patient?
a. Discussion of the benefits of breastfeeding as an excellent long term form of birth control.
b. Cervical swabs for N. Gonorrhoeae and C. Trachomatis.
c. Prescription for a combined oral contraceptive pill.
d. Prophylactic sertraline for postpartum depression.
e. Obtaining consent for the administration of depo medroxyprogesterone acetate
during this visit.

A

The best answer here is E.
Progestin only contraceptives are the ideal option

How well did you know this?
1
Not at all
2
3
4
5
Perfectly
147
Q

In the postpartum period, ——— may be ok as birth control for a 6 mo period, however it is not completely reliable.

A

breastfeeding

How well did you know this?
1
Not at all
2
3
4
5
Perfectly
148
Q

In a woman that expresses a desire for birth control postpartum, resist the temptation to prescribe

A

anything containing estrogen (inhibits breastfeeding)

How well did you know this?
1
Not at all
2
3
4
5
Perfectly
149
Q

Progestin only contraceptives MOA involves

A

thickening cervical mucus

How well did you know this?
1
Not at all
2
3
4
5
Perfectly
150
Q

Progestin only contraceptives protect against

A

endometrial cancer

How well did you know this?
1
Not at all
2
3
4
5
Perfectly
151
Q

Progestin only contraceptives avoided in the setting of

A

breast cancer (or other progesterone receptor positive gynecologic malignancies)

How well did you know this?
1
Not at all
2
3
4
5
Perfectly
152
Q

Association of injectable progesterone with (3)

A

delayed return to fertility after discontinuation (up to a year)

possible weight gain

in some cases, a decrease in bone mineral density

How well did you know this?
1
Not at all
2
3
4
5
Perfectly
153
Q

Treat breast mastitis with

A

dicloxacillin

How well did you know this?
1
Not at all
2
3
4
5
Perfectly
154
Q

A 26-year-old patient who is 4-weeks postpartum presents with a 1-day history of fever of 100.9°F (38.3°C) and breast tenderness. She has been breast-feeding without problems and reports no other symptoms. Her left breast has a 4-cm area of induration and erythema at the 3 o’clock position that is tender to palpation. Milk expressed from that breast is white. What is the most likely diagnosis? What is the treatment?

A

Mastitis. Focal area of breast infection and fever approximately 1 month postpartum is consistent with mastitis. The patient should be started on dicloxacillin.

How well did you know this?
1
Not at all
2
3
4
5
Perfectly
155
Q

Breast-Associated Fevers: Mastitis timeframe

A
How well did you know this?
1
Not at all
2
3
4
5
Perfectly
156
Q

Breast-Associated Fevers: Mastitis timeframe

A
How well did you know this?
1
Not at all
2
3
4
5
Perfectly
157
Q

Mastitis (Breast-Associated Fevers): timeframe

A

Anytime while lactating (usually >3–4 weeks postpartum)

How well did you know this?
1
Not at all
2
3
4
5
Perfectly
158
Q

Mastitis (Breast-Associated Fevers): presentation

A

Focal erythema and induration, streaking on the breast

How well did you know this?
1
Not at all
2
3
4
5
Perfectly
159
Q

Mastitis (Breast-Associated Fevers): causative agent

A

Staph aureas, MRSA, coag negative staph

How well did you know this?
1
Not at all
2
3
4
5
Perfectly
160
Q

Mastitis (Breast-Associated Fevers): management

A

Treat with dicloxacillin for 7–10 days

How well did you know this?
1
Not at all
2
3
4
5
Perfectly
161
Q

Mastitis (Breast-Associated Fevers): continue breastfeeding?

A

Yes

How well did you know this?
1
Not at all
2
3
4
5
Perfectly
162
Q

Breastfeeding is associated with a decreased risk of (2)

A

Breast cancer (Prolactin shuts down GnRH which nukes estrogen production. Estrogen drives many breast cancers.)

Ovarian cancer (Prolactin shuts down GnRH which causes anovulation. With anovulation, the ovarian epithelium does not have to be broken down and repaired each month (which happens with ovulation). This reduces the potential for malignant transformation of ovarian epithelium.)

How well did you know this?
1
Not at all
2
3
4
5
Perfectly
163
Q

Breast feeding helps with

A

weight loss

How well did you know this?
1
Not at all
2
3
4
5
Perfectly
164
Q

A woman with (3) should not breastfeed

A

HIV

active TB

active herpes lesions on the breast

How well did you know this?
1
Not at all
2
3
4
5
Perfectly
165
Q

Infants with ——— is a contraindication to breastfeeding.

A

Galactosemia

How well did you know this?
1
Not at all
2
3
4
5
Perfectly
166
Q

A 28-year-old woman, gravida 1, para 1, comes to the office to discuss a recent cervical pap smear. The cytology report says that she has atypical squamous cells of undetermined significance (ASCUS). She has never had an abnormal pap smear result before. Reflex human papillomavirus (HPV) testing is negative. Which of the following is the most appropriate next step in managing this patient?
- Human papillomavirus vaccine
- Repeat pap smear with HPV testing in 1 year
- Repeat pap smear with HPV testing in 3 years
- Colposcopy with biopsies
- Loop electrosurgical excision procedure

A

Repeat pap smear with HPV testing in 3 years (Note: At which point 28yo pt will be over 30!)

Co-testing, pap smear and human papillomavirus (HPV) testing, is an appropriate follow-up plan for young women with atypical squamous cells of undetermined significance (ASCUS). This should be performed 3 years after the abnormal cytology report.

For the woman described in the question, a pap smear has identified atypical cells of undetermined significance (ASCUS). According to the American Society for Colposcopy and Cervical Pathology (ASCCP) as well as the American College of Obstetricians and Gynecologists (ACOG), the preferred next step is to do reflex human papillomavirus (HPV) testing. If HPV testing is negative, as in this patient, the next step is to repeat a pap smear with reflex HPV testing in 3 years (this is known as “co-testing”). However, if HPV testing is positive, colposcopy is warranted. For women age 30 or great, ASCUS doesn’t even warrant HPV testing - current recommendations are just to do co-testing in 3 years.
An acceptable alternative to reflex HPV testing is to repeat the pap test in one year, but HPV testing is more likely to grant immediate peace of mind, as well as reduce the number of procedures performed.

How well did you know this?
1
Not at all
2
3
4
5
Perfectly
167
Q

A 30-year-old woman is brought to the emergency department because of altered mental status. Collateral history shows that the patient has been fatigued over the last 24 hours, and today did not recognize her husband. In addition, she has had a decreased appetite and vomiting over the past 12 hours. She is known to have cervical cancer for which she is awaiting treatment. Current medications include only oxycodone. Vitals signs show her temperature is 37°C (98.6°F), pulse is 85/min, respirations are 26/min, blood pressure is 160/90 mmHg, and oxygen saturation is 94% on room air. Physical examinations shows a drowsy, diaphoretic female, asterixis, and moderate distension inferior to the umbilicus. Initial laboratory results are shown below.
Sodium: 135 mEq/L
Potassium: 4.9 mEq/L
Chloride: 99 mmol/L
Bicarbonate: 12 mEq/L
Blood Urea Nitrogen: 65 mg/dL (high)
Creatinine: 3.5 mg/dL (high)
Glucose: 122 mg/dL
Hemoglobin: 14 g/dL
Which of the following is the most likely cause of this patient’s altered mental status?
- Medication overdose
- Uremic encephalopathy
- Brain metastasis
- Hepatic encephalopathy
- Severe sepsis

A

Uremic encephalopathy

Complications of malignancy can present in a variety of ways, as shown in this case of cervical cancer. Obstructive uropathy can cause post-renal failure leading to uremia and uremic encephalopathy. The patient has confusion, decreased alertness, vomiting, and asterixis, all which are consistent with an encephalopathic condition.

This patient has confusion, decreased alertness, vomiting, and asterixis, all of which are consistent with an encephalopathic condition. Note that the patient also has acute renal failure, with a BUN of 65 g/dL and a creatinine of 3.5 mg/dL, and an anion gap of 24. The most likely cause of these abnormalities is acute uremic encephalopathy. Uremia will cause an anion gap metabolic acidosis, which may explain why the patient is diaphoretic and tachypneic. One possible etiology of this patient’s renal failure is obstructive uropathy from the underlying cervical cancer compressing the lower urinary outflow tract. The patient’s lower abdominal distension likely represents a distended bladder.

How well did you know this?
1
Not at all
2
3
4
5
Perfectly
168
Q

HPV types associated with low-
grade cervical lesions and 90% of anogenital warts

A

Types 6 and 11

How well did you know this?
1
Not at all
2
3
4
5
Perfectly
169
Q

HPV types associated with >50% of high-grade cervical lesions and 70% of cervical cancer

A

HPV Types 16 and 18

How well did you know this?
1
Not at all
2
3
4
5
Perfectly
170
Q

Vaccines against high-risk strains of HPV are currently FDA approved for

A

females and males between the ages of 9 and 45 years

How well did you know this?
1
Not at all
2
3
4
5
Perfectly
171
Q

Risk Factors for Cervical Dysplasia
and Cervical Cancer

A

■ HPV infection
■ ↑ sexual activity (↑ risk of viral/bacterial infections)
■ Low socioeconomic status (likely due to limited access to health care and screening)
■ Genetic predisposition
■ Cigarette smoking (increases risk for squamous cell cancers but not adenocarcinomas)
■ OCPs (condoms
↓ risk in these women)
■ Immunosuppression

How well did you know this?
1
Not at all
2
3
4
5
Perfectly
172
Q

A 21-year-old G2P2 patient desires contraception. She has been sexually active for
4 years, with three lifetime partners. Her menses are irregular. Before prescribing combined OCPs, what tests need to be performed?

A

A pregnancy test, Pap test, and STI screening

How well did you know this?
1
Not at all
2
3
4
5
Perfectly
173
Q

Define a Pap test

A

A Pap test is a cytologic screening test for cervical dysplasia

How well did you know this?
1
Not at all
2
3
4
5
Perfectly
174
Q

The methods available for cervical cancer screening are

A

Pap test (i.e., cytology), HPV testing, and co-testing with both cytology and HPV

How well did you know this?
1
Not at all
2
3
4
5
Perfectly
175
Q

The ectocervix is comprised of ——— epithelium, and the endocervix (including cervical canal) is comprised of ——— epithelium.

A

squamous

glandular (columnar)

How well did you know this?
1
Not at all
2
3
4
5
Perfectly
176
Q

Pap test is a screening test that provides ——— information, not ——— information

A

cytologic

histologic

How well did you know this?
1
Not at all
2
3
4
5
Perfectly
177
Q

Pap test screening should begin

A

not begin until age 21, regardless of sexual activity

How well did you know this?
1
Not at all
2
3
4
5
Perfectly
178
Q

USPSTF recommendations on screening for cervical cancer

A

Screening in women age 21 to 65 years with cytology (Pap smear) every 3 years or, for women ages 30 to 65 years who want to lengthen the screening interval, screening with a combination of cytology and HPV testing every 5 years

How well did you know this?
1
Not at all
2
3
4
5
Perfectly
179
Q

Pap smears are not helpful in women after

A

hysterectomy (for non-cancer disease)

How well did you know this?
1
Not at all
2
3
4
5
Perfectly
180
Q

Contrast types of cervical cancer in terms of commonness and location:

A

Squamous cell cancer (outer cervix) is the most common kind of cervical cancer

Adenocarcinomas can also occur but are typically in the inner cervix that is not necessarily visible with a speculum exam

How well did you know this?
1
Not at all
2
3
4
5
Perfectly
181
Q

Most cervical cancers arise in the

A

transformation zone between the ecto and endocervix

(Specifically, most cervical cancers arise at the squamocolumnar junction)

How well did you know this?
1
Not at all
2
3
4
5
Perfectly
182
Q

RFs for cervical cancer:

A

The biggest RF is a history of exposure to HPV (especially 16 and 18).

Other RFs include:
having multiple sex partners, having a h/o of STDs,
having HIV (immunosuppression),
smoking

How well did you know this?
1
Not at all
2
3
4
5
Perfectly
183
Q

You certainly need to know the screening guidelines for cervical cancer.

A

A woman should be screened every 3 years from 21-29. A similar regimen can be pursued after the age of 30. However, a pap smear + HPV screening can be conducted every 5 years. Screening can be stopped at 65 if multiple pap smears have been normal.

How well did you know this?
1
Not at all
2
3
4
5
Perfectly
184
Q

Screening for cervical cancer no longer necessary if:

A

A patient that undergoes a hysterectomy FOR BENIGN REASONS does not need further screening after the procedure. For NON-BENIGN reasons, a pap smear of the vaginal cuff must be undertaken at routine intervals.

How well did you know this?
1
Not at all
2
3
4
5
Perfectly
185
Q

A pap smear revealing ASCUS (atypical squamous cells of undetermined significance) should be followed by ——— if the patient is over 25. If HPV+, ———. If < 25, ———. If +ve, ———.

A

HPV testing

proceed to a colposcopy

an acceptable option is to repeat the pap in a year (but preferred from osmosis also HPV Testing)

proceed to colposcopy
Option 1 (for the > 25 yo F) is also acceptable.

How well did you know this?
1
Not at all
2
3
4
5
Perfectly
186
Q

If a pap smear reveals ASC-H (ASC but cannot exclude HSIL; atypical squamous cells, cannot exclude a high-grade squamous intraepithelial lesion):

A

proceed to a colposcopy

How well did you know this?
1
Not at all
2
3
4
5
Perfectly
187
Q

If a pap smear reveals AGUS (Atypical Glandular Cells of Undetermined Significance), your NBSIM is:

A

to get a whole bunch of tests:
colposcopy, endocervical curettage, and an an endometrial biopsy

How well did you know this?
1
Not at all
2
3
4
5
Perfectly
188
Q

If a pap smear reveals low-grade squamous intraepithelial lesions (LSIL or cervical intraepithelial neoplasia (CIN 1)), it is typically acceptable to ———. In some rare cases with super low risk individuals (aka really young), ———

A

get a colposcopy with biopsy

the pap may be repeated in a year since these individuals will more than likely clear the infection

How well did you know this?
1
Not at all
2
3
4
5
Perfectly
189
Q

High-grade squamous intraepithelial lesions (HSIL or CIN 2 or 3),———l

A

proceed to a colposcopy always

An unusual answer you may see on the NBME is to perform a Loop Electrosurgical Excision Procedure (LEEP) procedure to excise the lesion

How well did you know this?
1
Not at all
2
3
4
5
Perfectly
190
Q

If a pap smear shows LSIL (CIN 1) but a colposcopy with biopsy shows something different (e.g. HSIL with CIN 2⁄3):

A

repeat the colposcopy and get an endocervical curettage

How well did you know this?
1
Not at all
2
3
4
5
Perfectly
191
Q

Consider cervical cancer if a Q stem gives you a patient with cervical cancer RFs in the setting of:

A

post-coital bleeding

How well did you know this?
1
Not at all
2
3
4
5
Perfectly
192
Q

Cervical cancer can metastasize to ——— structures and cause ———

A

urinary system (like ureters)

hydronephrosis leading to renal failure (When cervical cancer spreads to other areas of the pelvis, it can block one or both ureters, which carry urine from the kidneys to the bladder. This can lead to a condition called hydronephrosis. In hydronephrosis, blocked ureters cause urine to build up in the kidneys.) This is the MCCOD in cervical cancer.

How well did you know this?
1
Not at all
2
3
4
5
Perfectly
193
Q

Key complication of LEEP and conization procedures:

A

cervical insufficiency (painless pregnancy loss)

How well did you know this?
1
Not at all
2
3
4
5
Perfectly
194
Q

A 22-year-old woman comes to her primary care physician’s office for a follow up of an abnormal Pap smear result. The patient had a Pap smear completed one month ago, which showed a low-grade squamous intraepithelial lesion, and tested positive for double stranded DNA naked icosahedral DNA virus. History reveals she began having sexual intercourse at age 16 and has had 4 lifetime partners. There is no history of sexually transmitted infections or pregnancies. She uses oral contraceptive pills regularly and condoms “most of the time.” She denies tobacco or illegal drug use. She has not had any vaccinations for the prevention of sexually transmitted infections. Which of the following is the most appropriate next step of management in this patient?
- Loop electrosurgical excision procedure
- Hysterectomy
- Repeat cervical cytology in 12 months
- Repeat Pap smear in 4 months
- Colposcopy

A

Repeat cervical cytology in 12 months

HPV, double stranded DNA virus, is the most commonly transmitted sexual infection and is a major cause of cervical cancer. Patients ages 21-24 with a low-grade squamous intraepithelial lesion should be treated conservatively and scheduled for a follow up cytologic screening test in 12 months

How well did you know this?
1
Not at all
2
3
4
5
Perfectly
195
Q

Cytologic (Pap test) abnormalities are described using the term ———. Histologic (biopsy) abnormalities are described using the term ———.

A

“squamous intraepithelial lesion” (SIL)

“cervical intraepithelial neoplasia” (CIN)

How well did you know this?
1
Not at all
2
3
4
5
Perfectly
196
Q

A 45-year-old G4P4 patient presents for a routine annual exam. Her Pap test returns
with a report of ASCUS. Her Pap tests have always been normal in the past. What is the next best step to evaluate her cancer risk?

A

High-risk HPV DNA testing. If high-risk HPV DNA testing is “positive,” indicating the presence of a hrHPV strain, then a colposcopy and indicated biopsies should be performed. If the HPV testing is negative, she can be managed as per her age-based protocol.

How well did you know this?
1
Not at all
2
3
4
5
Perfectly
197
Q

A 35-year-old woman comes to the office because of irregular bleeding since her last period three weeks ago. She says her bleeding is consistently postcoital. Examination shows no abdominal masses and no peripheral edema. Her urethra is normal, and her fecal occult blood test shows no abnormalities. She is not pregnant. Which of the following is the most likely diagnosis?
-Uterine fibroids
-Urethral caruncle
-Ectopic pregnancy
-Peri-menopausal bleeding
-Cervical neoplasia

A

Cervical neoplasia may be characterized by postcoital bleeding for about 11% of cases. Postcoital bleeding refers to spotting or bleeding unrelated to menstruation that occurs during or after sexual intercourse. Other causes include cervical polyps, vaginal cancer, or genital infections.
(If the bleeding is consistently postcoital, it suggests cervical pathology, such as cervical polyps, cervicitis, or cervical neoplasia.)

How well did you know this?
1
Not at all
2
3
4
5
Perfectly
198
Q

ACOG Screening Guidelines for cervical cancer in individuals aged 21–29

A

Individuals aged 21–29 should be screened with cervical cytology (Pap test) alone every 3 years

How well did you know this?
1
Not at all
2
3
4
5
Perfectly
199
Q

ACOG Screening Guidelines for cervical cancer in individuals aged 30–65

A

There are now three recommended options for cervical cancer screening
in individuals aged 30–65 years:
■ Cytology alone every 3 years.
■ FDA-approved primary hrHPV testing alone every 5 years.
■ Co-testing with cytology and hrHPV testing every 5 years.

How well did you know this?
1
Not at all
2
3
4
5
Perfectly
200
Q

ACOG Screening Guidelines for cervical cancer in high risk individuals

A

Frequency of Pap test screening should be individualized, and may need to occur earlier/more often in women who have HIV, are immune compromised, were exposed to diethylstilbestrol (DES) in utero, or have been previously treated for CIN2, CIN3, or cancer.

How well did you know this?
1
Not at all
2
3
4
5
Perfectly
201
Q

ACOG Screening Guidelines for cervical cancer in women under age 21

A

Women under age 21 should not be screened, because the incidence of cervical cancer is very low, and they have an effective immune response that will usually clear the HPV infection in 8–24 months.

How well did you know this?
1
Not at all
2
3
4
5
Perfectly
202
Q

ACOG Screening Guidelines for cervical cancer in women after age 65

A

Pap tests may be discontinued after age 65 in healthy women with adequate negative prior screening test (Pap test) results and no history of CIN2 or higher.

How well did you know this?
1
Not at all
2
3
4
5
Perfectly
203
Q

ACOG Screening Guidelines for cervical cancer in women after hysterectomy

A

A Pap test may be discontinued after hysterectomy (with removal of the cervix) for benign disease.

How well did you know this?
1
Not at all
2
3
4
5
Perfectly
204
Q

A 15-year-old girl comes to the office for a routine annual physical examination. During the appointment, her mother inquires about the human papillomavirus (HPV) vaccine and is happy for her daughter to get the vaccine today. When her mother steps out of the office, the patient says that she started having sex with her boyfriend about two months ago, and she does not want her mother to know. They have always used condoms and her last menstrual period was eight days ago. Which of the following is the most appropriate next step in management?
-Inform the patient’s mother that she cannot get the vaccine because she is already sexually active
-Do a human papillomavirus test to make sure she has not been infected before she gets the vaccine
-Administer a dose today, a second shot in two months, and a third shot in six months
- Administer the vaccine anyway, but it will not be effective
- Obtain a pregnancy test before administering the vaccine

A
  • Administer a dose today, a second shot in two months, and a third shot in six months
    The quadrivalent human papillomavirus (HPV) vaccine protects against HPV 6 and 11, which cause 90% of genital warts, and HPV 16 and 18, which cause 70% of cervical cancer cases. Although it is most efficient prior to the onset of sexual activity, it is still recommended for people who are already sexually active.

The human papillomavirus (HPV) vaccine is a three-dose series that is currently recommended for girls and boys ages 9-26 (but ideally between 11-12 years of age) to prevent infection with HPV. The quadrivalent vaccine protects against HPV 6 and 11, which cause 90% of genital warts, and HPV 16 and 18, which cause 70% of cervical cancer cases. There is also a bivalent vaccine which only covers HPV 16 and 18.

The vaccine has a very good safety profile, and few side effects beyond tenderness at the injection site have been noted. Although the human papillomavirus (HPV) vaccine is contraindicated in pregnancy, routine pregnancy testing is not recommended prior to vaccination. The patient is using contraceptives and her last menstrual period was a week ago, making pregnancy unlikely. She should be urged to continue using contraceptives for the duration of the series. The vaccine is contraindicated in pregnancy because there have not been enough studies of its safety in this group (pregnancy category B).
However, in women who were inadvertently vaccinated without prior knowledge of their pregnancy status, there has been no increased risk to the fetus, and there is no need to terminate such a pregnancy. The vaccine series can be completed after the pregnancy.

How well did you know this?
1
Not at all
2
3
4
5
Perfectly
205
Q

What HPV genotypes are contained in the Gardasil quadrivalent vaccine?

A

Types 6, 11, 16, and 18

Protects against 70% of cervical cancers and 90% of genital warts

(Note: Only the 9-valent vaccine is available in the United States (GARDASIL-9))

How well did you know this?
1
Not at all
2
3
4
5
Perfectly
206
Q

Administration schedule for HPV vaccine (Gardasil)

A

Administered as three doses: 0.5 mL intramuscularly given at intervals of 0, 2, and 6 months

How well did you know this?
1
Not at all
2
3
4
5
Perfectly
207
Q

US Advisory Committee on Immunization Practices (ACIP) recommendations for HPV Vaccines:

A
  1. Routine vaccination at age 11–12, but can be given as early as age 9.
  2. For adolescents and adults aged 13–26 years who have not been previously vaccinated or have not completed the series, catch-up vaccination is recommended.
  3. For adults 27 years and older, catch-up vaccination is not routinely recommended, and should be made on an individual basis.

Not recommended for pregnant women. Can be given to breast-feeding women

How well did you know this?
1
Not at all
2
3
4
5
Perfectly
208
Q

Management of abnormal Pap tests in women ages ——— differs from those age ——— because of the low incidence of cervical cancer in this age group.

A

21–24

≥25

How well did you know this?
1
Not at all
2
3
4
5
Perfectly
209
Q

CIN1 is a ———. It is referred to as ——— in the LAST system.

A

low-grade lesion with mild atypical changes in the lower one-third of the epithelium

low-grade SIL (LSIL)

How well did you know this?
1
Not at all
2
3
4
5
Perfectly
210
Q

CIN2 is a ———. Specimens that are p16-negative are referred to as ——— and those that are p16-positive are referred to as ———

A

high-grade lesion, with atypical changes confined to the lower two-thirds of the epithelium (difficult to differentiate from CIN3; stratified according to p16 immunostaining to identify precancerous lesions)

LSIL

high-grade SIL (HSIL)

How well did you know this?
1
Not at all
2
3
4
5
Perfectly
211
Q

CIN3 is a ———, and is referred to as ——— in the LAST system.

A

high-grade lesion, where atypical cells encompass >2/3 of the epithelium

HSIL

(CIN2 and CIN3 are often treated the same)

How well did you know this?
1
Not at all
2
3
4
5
Perfectly
212
Q

A 45-year-old G4P4 patient presents for a routine annual exam. Her Pap test returns
with a report of ASCUS. Her Pap tests have always been normal in the past. What is the next best step to evaluate her cancer risk?

A

High-risk HPV DNA testing. If high-risk HPV DNA testing is “positive,” indicating the presence of a hrHPV strain, then a colposcopy and indicated biopsies should be performed. If the HPV testing is negative, she can be managed as per her age-based protocol.

How well did you know this?
1
Not at all
2
3
4
5
Perfectly
213
Q

Next step if pap finds atypical squamous cells of undetermined significance (ASCUS):

A
  1. Reflex HPV testing if under age 30, concurrent HPV testing if age 30 and above.
  2. If HPV negative—continue routine screening per protocol.
  3. If HPV positive—colposcopy if age ≥25, repeat cytology (Pap test) in 1 year if age 21–24.
How well did you know this?
1
Not at all
2
3
4
5
Perfectly
214
Q

Next step if pap finds atypical squamous cells, cannot exclude HSIL (ASC-H):

A

Colposcopy with indicated biopsies

How well did you know this?
1
Not at all
2
3
4
5
Perfectly
215
Q

Next step if pap finds LSIL:

A

Colposcopy with indicated biopsies if age ≥25, repeat cytology (Pap test) in 1 year if age 21–24

How well did you know this?
1
Not at all
2
3
4
5
Perfectly
216
Q

Next step if pap finds HSIL:

A

Colposcopy with endocervical curettage and indicated biopsies

How well did you know this?
1
Not at all
2
3
4
5
Perfectly
217
Q

List results of an abnormal Pap test in order of severity:

A

●Atypical squamous cells of undetermined significance (ASC-US)
●Low-grade squamous intraepithelial lesions (LSIL)
●Atypical squamous cells cannot exclude high-grade squamous intraepithelial lesion (ASC-H)
●High-grade squamous intraepithelial lesions (HSIL)
●Atypical glandular cells (AGC)

How well did you know this?
1
Not at all
2
3
4
5
Perfectly
218
Q

Next step if pap finds atypical glandular cells (AGC):

A

Colposcopy with indicated biopsies, endocervical curettage (ECC), and endometrial biopsy

How well did you know this?
1
Not at all
2
3
4
5
Perfectly
219
Q

——— percent of women with abnormal cytologic findings can be adequately evaluated with colposcopy.

A

Ninety

How well did you know this?
1
Not at all
2
3
4
5
Perfectly
220
Q

What must be completely visualized for adequate colposcopic evaluation?

A
  1. TZ
  2. Extent of lesion in its entirety
How well did you know this?
1
Not at all
2
3
4
5
Perfectly
221
Q

Define Colposcopy with Cervical Biopsy

A

A procedure that utilizes staining and a low-magnification microscope, mounted on a stand, for the viewing of the cervix, vagina, and vulva; illuminated, magnified view, which aids in identifying lesions and biopsying suspicious areas to obtain histologic diagnosis.

(1. Speculum inserted; 2. Acetic acid applied: The neoplastic cells appear whiter; 3. Colposcopy; 4. Cervical biopsy: Neoplastic and dysplastic areas are then biopsied under colposcopic guidance; 5. ECC: A curette is used to scrape the cervical canal to obtain endocervical cells for cytologic examination.

How well did you know this?
1
Not at all
2
3
4
5
Perfectly
222
Q

If colposcopy-based biopsy results or ECC is positive for CIN2 or CIN3 (HSIL), then

A

a cone biopsy or a LEEP should be performed as both a diagnostic and therapeutic procedure

How well did you know this?
1
Not at all
2
3
4
5
Perfectly
223
Q

A 23-year-old woman, gravida 1, para 1, comes to the office to discuss a recent cervical pap smear from her annual examination. The cytology report says that she has a high-grade squamous intraepithelial lesion (HSIL). She has no complaints and none of her previous cytology reports show any abnormalities. Which of the following tests or treatments is the best next step in managing this patient?
-Return in 6 months for repeat pap smear
-Human papillomavirus vaccine
-Return in 3 months for repeat pap smear
-Colposcopy with biopsies
-Loop electrosurgical excision procedure

A

Colposcopy with biopsies
Colposcopy and biopsy of any suspected lesions is the best management plan for a woman aged 21-24 years old with a cytological diagnosis of high-grade squamous intraepithelial lesion (HSIL). In women >25 years old, either colposcopy or loop electrosurgical excision procedure (LEEP) would be an appropriate recommendation.

How well did you know this?
1
Not at all
2
3
4
5
Perfectly
224
Q

A 25-year-old woman comes to the clinic for a Pap smear. She has been treated for multiple sexually transmitted infections in the past. She is sexually active and does not use contraception. Her temperature is 37.0°C (98.6°F), pulse is 80/min, and respirations are 14/min. Physical examination shows no abnormalities. Pelvic examination is normal without evidence of cervical discharge or motion tenderness. A Pap smear is obtained and shows high-grade squamous intraepithelial lesions. Colposcopy is performed and shows cervical intraepithelial neoplasia (CIN) grade 3. Which of the following is the most appropriate treatment for this patient’s condition?
-Abdominal hysterectomy
-Repeat colposcopy to rule out false positive
-Vaginal hysterectomy
-High dosage of prednisone
-Loop electrical excision procedure

A

Loop electrical excision procedure
In patients who have cervical intraepithelial neoplasia (CIN) grade 3, a cervical ablative procedure is the next best step in management. This can be accomplished with a loop electrical excision procedure (LEEP).

Excisional or ablative procedures, such as a loop electrical excision procedure (LEEP), laser therapy, cryotherapy, and cervical cone biopsy are appropriate for the management of cervical intraepithelial neoplasia (CIN) grades 2 or 3.
CIN 1 lesions often regress spontaneously without further management and only follow up for repeat smear or colposcopy is required. The rate of spontaneous regression of CIN 2 or 3 is lower, warranting a procedure to remove the abnormal cells. If left untreated, CIN may progress to cervical cancer.

How well did you know this?
1
Not at all
2
3
4
5
Perfectly
225
Q

Cone Biopsy and Loop Electrosurgical Excision Procedure (LEEP) are both excisional procedures which involve

A

excising a cone-shaped portion of the cervix, including the endocervical canal and TZ

(They are intended to be both diagnostic and therapeutic)

How well did you know this?
1
Not at all
2
3
4
5
Perfectly
226
Q

A 24-year-old woman, gravida 0, para 0, comes to the office for a routine gynecologic examination. She has no specific complaints. Her first pap smear from three years ago showed normal cervical cytology. Last year, she received 3 doses of the human papillomavirus (HPV) vaccine. She is sexually active with two male partners and says that they use condoms intermittently. She is otherwise in good health and her medical history is noncontributory. Her paternal grandmother was diagnosed with cervical cancer at age 55. Which of the following is the most appropriate option for the patient at this time?
- Do not perform pap smear at this visit; women do not need routine cervical cytology screening if they have received the human papillomavirus vaccine
- Perform pap smear at this visit; women 21-29 years old should receive cervical cytology screening every 3 years
- More information about the patient’s family history of cervical cancer is needed before a decision can be made about cervical cytology screening
- Perform pap smear at this visit; women with multiple sexual partners should receive cervical cytology screening more frequently than the general population
- Do not perform pap smear at this visit; women aged 21-29 years old should receive combined cervical cytology screening and HPV co-testing every 5 years

A

Perform pap smear at this visit; women 21-29 years old should receive cervical cytology screening every 3 years

Cervical cytology screening (pap smear) for cervical cancer is recommended every 3 years for women aged 21-29, and cervical cytology plus HPV co-testing every 5 years in women aged 30-65 by the American College of Obstetricians and Gynecologists.

How well did you know this?
1
Not at all
2
3
4
5
Perfectly
227
Q

Indications for cone biopsy/leep:

A

■ Inadequate view of TZ on colposcopy
■ Positive ECC
■ Treatment for CIN2–CIN3 (HSIL)
■ Treatment for adenocarcinoma in situ
■ When cancer cannot be excluded after colposcopy, biopsy, and ECC

How well did you know this?
1
Not at all
2
3
4
5
Perfectly
228
Q

Evaluation of ——— may be challenging with LEEP, because of thermal artifact.

A

biopsy margins

How well did you know this?
1
Not at all
2
3
4
5
Perfectly
229
Q

A 25-year-old G1P0 patient at 38 weeks’ gestation presents to triage reporting contractions that have been increasing in strength and frequency over a 12-hour period. She does not have vaginal bleeding, leakage of fluid, or preeclampsia symptoms. She reports good fetal movement. Fetal heart rate (FHR) is reassuring. She is contracting every 2 minutes on the monitor. The cervical exam is 6 cm dilated, 50% effaced, 0 station, cephalic. What stage of labor is she in? If her labor progresses as expected, what should her cervical dilation be at the next vaginal exam (VE) in 2 hours?

A

She is in the active phase of the first stage of labor. Since she is a primigravida, her cervix should dilate at a minimum of 1.2 cm/hr. So, in 2 hours, she should be 8.4 cm (or 8–9 cm) dilated.

How well did you know this?
1
Not at all
2
3
4
5
Perfectly
230
Q

Prelabor rupture denotes:

A

spontaneous rupture of fetal membranes before the onset of labor
(Ie, ROM: Rupture of membranes vs PROM: Prelabor rupture of membranes (ROM before the onset of labor))

(Premature ROM can occur at term (PROM) or preterm (PPROM: <37 weeks))

How well did you know this?
1
Not at all
2
3
4
5
Perfectly
231
Q

Most common diagnosis associated with preterm delivery

A

PPROM

How well did you know this?
1
Not at all
2
3
4
5
Perfectly
232
Q

Prolonged rupture of membranes refers to

A

a rupture of membranes lasting longer than 18 hours (i.e. between time of rupture and time of delivery)

How well did you know this?
1
Not at all
2
3
4
5
Perfectly
233
Q

Duration of labor is typically shorter in

A

the multiparous patient than in nulliparous patients

How well did you know this?
1
Not at all
2
3
4
5
Perfectly
234
Q

Labor is defined as

A

regular contractions that result in cervical change

(A patient can have contractions that do not cause cervical change as well as cervical change without contractions—neither of these are “labor”)

How well did you know this?
1
Not at all
2
3
4
5
Perfectly
235
Q

Number of stages of labor

A

There are three stages of labor and two phases of stage 1

How well did you know this?
1
Not at all
2
3
4
5
Perfectly
236
Q

Describe first stage of labor:

A

Begins with onset of uterine contractions of sufficient frequency, intensity, and duration to result in effacement and dilation of the cervix, and ends when the cervix is completely dilated to 10 cm.
Consists of two phases:
1. Latent phase: Begins with the onset of labor and ends at approximately 4–6 cm cervical dilation.
2.Active phase: Rapid dilation. Ends at 10cm.

How well did you know this?
1
Not at all
2
3
4
5
Perfectly
237
Q

Describe the second stage of labor

A

the stage of fetal expulsion:
It begins when the cervix is fully dilated and ends with the delivery of the fetus

How well did you know this?
1
Not at all
2
3
4
5
Perfectly
238
Q

Describe the third stage of labor:

A

The main event is placental separation: It begins immediately after the delivery of the fetus and ends with the delivery of the fetal and placental membranes
(Duration: Usually <10 minutes; considered prolonged if >30 minutes)

How well did you know this?
1
Not at all
2
3
4
5
Perfectly
239
Q

The three signs of placental separation are:

A
  1. Gush of blood from vagina
  2. Umbilical cord lengthening
  3. Fundus of the uterus rises up and becomes firm
    Know these! They are commonly asked at delivery.
How well did you know this?
1
Not at all
2
3
4
5
Perfectly
240
Q

Remember the three “Ps” that affect the duration of the active phase of labor:

A

■ Power (strength and frequency of contractions)
■ Passenger (size of the baby)
■ Pelvis (size and shape of mother’s pelvis)

How well did you know this?
1
Not at all
2
3
4
5
Perfectly
241
Q

If progress during the active phase is slower than expected, evaluation for adequacy of ———should be done.

A

uterine contractions, fetal malposition, or cephalopelvic disproportion (CPD)

How well did you know this?
1
Not at all
2
3
4
5
Perfectly
242
Q

Abnormalities of the second stage may be either:

A

protraction or arrest of descent (the fetal head descends <1 cm/hr in a nullipara and <2 cm/hr in a multipara)

How well did you know this?
1
Not at all
2
3
4
5
Perfectly
243
Q

In third stage of labor, if 30 minutes have passed without placental expulsion, ——— may be required

A

manual removal of the placenta

How well did you know this?
1
Not at all
2
3
4
5
Perfectly
244
Q

What are the three signs of placental separation?

A
  1. Gush of blood
  2. Umbilical cord lengthening
  3. Fundus of uterus rises and firms
How well did you know this?
1
Not at all
2
3
4
5
Perfectly
245
Q

Contrast true vs false labor in terms of:
Regularity of intervals
Intensity
Location of discomfort
Impact on cervix
Effect of medications

A
How well did you know this?
1
Not at all
2
3
4
5
Perfectly
246
Q

Bloody show is

A

small amount of blood mixed with cervical mucus that is present with cervical dilation and effacement
(should be distinguished from vaginal bleeding)

How well did you know this?
1
Not at all
2
3
4
5
Perfectly
247
Q

Diagnosis of ROM: The patient’s history alone is correct in ——- of patients. ——— can be mistaken for ROM.

A

90%

Urinary leakage or excess vaginal discharge

How well did you know this?
1
Not at all
2
3
4
5
Perfectly
248
Q

Spontaneous rupture of membranes (SROM) most often occurs during the course of

A

active labor

How well did you know this?
1
Not at all
2
3
4
5
Perfectly
249
Q

A 25-year-old G1P0 patient at 39 weeks’ gestation presents to labor and delivery
reporting a gush of fluid from the vagina followed by constant leakage for 2 hours. The fluid is clear and without odor. What tests can help determine whether the patient has ruptured the membranes?

A

Sterile speculum exam, testing for pooling, ferning, and nitrazine. If these are positive, the membranes are likely ruptured and the fluid noted on the exam is likely amniotic fluid

How well did you know this?
1
Not at all
2
3
4
5
Perfectly
250
Q

Findings on sterile speculum exam indicative of ROM:

A
  1. Pooling: fluid collection in the posterior fornix or in the posterior blade of the speculum
  2. Valsalva: If pooling is not present, ask the patient to bear down and perform a Valsalva maneuver. Note if fluid is seen to come through the cervical os = Positive valsalva. Coughing can work here too.
  3. Ferning: Place a thin layer of the vaginal secretions or pooled fluid on a slide. View the dried amniotic fluid under a microscope for a characteristic ferning pattern made by the crystallized sodium chloride in the amniotic fluid (positive ferning). Confirms ROM in 85–98% of cases (see pic for true vs false fern and results when cervical mucous dries in the absence of amniotic fluid).
  4. Nitrazine: Place vaginal fluid on nitrazine paper to assess pH. If
    nitrazine paper turns blue, this indicates basic pH (positive nitrazine). Amniotic fluid has basic pH as compared to vaginal secretions that have acidic pH. Confirms ROM in 90–98% of cases.
How well did you know this?
1
Not at all
2
3
4
5
Perfectly
251
Q

What can cause a false-positive nitrazine test?

A

■ Vaginal infections with Trichomonas vaginalis or bacterial vaginosis
■ Blood
■ Semen

How well did you know this?
1
Not at all
2
3
4
5
Perfectly
252
Q

Define vernix:

A

The fatty substance consisting
of desquamated epithelial cells and sebaceous matter that normally covers the skin of the term fetus. More common in early term infants.

How well did you know this?
1
Not at all
2
3
4
5
Perfectly
253
Q

Define meconium

A

A dark green fecal material that collects in the fetal intestines and is discharged at or near the time of birth

How well did you know this?
1
Not at all
2
3
4
5
Perfectly
254
Q

Meconium staining is more common in

A

term and postterm pregnancies than in preterm pregnancies

How well did you know this?
1
Not at all
2
3
4
5
Perfectly
255
Q

Meconium aspiration syndrome (MAS): Define

A

Fetal stress, like hypoxia, leads to meconium in the amniotic fluid. With further fetal gasping, the meconium is inhaled into the fetal lungs, causing lung damage.

How well did you know this?
1
Not at all
2
3
4
5
Perfectly
256
Q

Meconium aspiration syndrome (MAS): Presentation

A

At birth, the infant will present with respiratory distress and can develop pulmonary hypertension.

How well did you know this?
1
Not at all
2
3
4
5
Perfectly
257
Q

Meconium aspiration syndrome (MAS): Tx

A

Intubation does not provide adequate oxygenation due to the lung injury and pulmonary hypertension. Infants with MAS may require extracorporeal membranous oxygenation (ECMO) which bypasses the lungs in order to provide oxygen to the baby

How well did you know this?
1
Not at all
2
3
4
5
Perfectly
258
Q

Cervix dilation describes ———and is determined by ———

A

Describes the size of the opening of the cervix at the internal os
(Ranges: From zero to 10 cm dilated)

Determination of dilation: index and/or the middle fingers are inserted in the cervical opening and are separated as far as the cervix will allow. The distance (cervical dilation) between the two fingers is estimated.

How well did you know this?
1
Not at all
2
3
4
5
Perfectly
259
Q

Cervical Effacement describes ———and is determined by ———

A

Describes the length of the cervix. With labor, the cervix thins out and softens, and the length is reduced (When the cervix shortens by 50% (to around 2 cm), it is said to be 50% effaced. When the cervix becomes as thin as the adjacent lower uterine segment, it is 100% effaced (think paper-thin)(

Determination of effacement: Palpate with finger and estimate the length from the internal to external os

How well did you know this?
1
Not at all
2
3
4
5
Perfectly
260
Q

Identify each labeled structure

A
How well did you know this?
1
Not at all
2
3
4
5
Perfectly
261
Q

A 33-year-old woman comes to the primary care clinic for an annual well-visit examination. She expresses a desire to become pregnant within the next year and inquires about steps she should take to ensure a healthy pregnancy. She has a past medical history of hypertension that is well-controlled with lisinopril. She has never been pregnant or has had prior issues with infertility. Temperature is 37 ºC (98.6 °F), heart rate is 66/min, blood pressure is 115/75 mmHg, and respiratory rate is 14/min. Physical examination is within normal limits. Which of the following is the most appropriate next step in management?
-Discontinue lisinopril and start nifedipine
-Discontinue lisinopril only after a positive pregnancy test
-Continue lisinopril as it is effectively controlling her blood pressure
-Increase lisinopril dosage to prevent hypertension-related complications in pregnancy
-Continue lisinopril and add oral folic acid supplementation

A

Discontinue lisinopril and start nifedipine
A key component of preconception counseling is to identify use of medications that are teratogenic. Renin-angiotensin system blocking agents are contraindicated in pregnancy and should be discontinued before conception. Patients should be switched to a pregnancy-safe antihypertensive like nifedipine, methyldopa, or labetalol.

This patient who is planning on becoming pregnant within the next year should be switched from her current ACE inhibitor to a pregnancy-safe antihypertensive like nifedipine. Agents that block the Renin-angiotensin system, such as ACE inhibitors and angiotensin receptor blockers (ARBs) are contraindicated in pregnancy and should be discontinued before conception.

How well did you know this?
1
Not at all
2
3
4
5
Perfectly
262
Q

ACE inhibitors and ARBs are contraindicated in pregnancy. These medications have been associated with

A

neonatal kidney failure and death

How well did you know this?
1
Not at all
2
3
4
5
Perfectly
263
Q

Antihypertensives that are safe to use in the perinatal period include

A

nifedipine, methyldopa, and labetalol

How well did you know this?
1
Not at all
2
3
4
5
Perfectly
264
Q

Patients with long-standing or uncontrolled hypertension who become pregnant should have ———. They should be counseled that they are at an increased risk of developing ——— in pregnancy, and they should be started on ——— to reduce their risk of preeclampsia.

A

vision screening, urine protein evaluation, and an electrocardiogram

preeclampsia and intrauterine growth restriction

low-dose aspirin after 12 weeks of gestation

How well did you know this?
1
Not at all
2
3
4
5
Perfectly
265
Q

Describe station

A

Describes the degree of descent of the presenting part in relation to ischial spines, which are designated at 0 station.

Terminology (two systems):
1. The ischial spine is zero station, and the areas above and below are divided into thirds. Above the ischial spines are stations –3, –2, and –1, with –3 being the furthest above the ischial spines and –1 being closest. Positive stations describe fetal descent below the ischial spines. +3 statian is at the level of the introitus, and +1 is just past the ischial spines.

  1. Very similar except that the areas above and below the ischial spines are divided by centimeters, up to 5 cm above and 5 cm below. Above are five stations or centimeters: –5, –4, –3, –2, and –1, with –5 being the 5 cm above the ischial spines and –1 being 1 cm above. Positive stations describe fetal descent below the ischial spines. +5 station is at the level of the introitus, and +1 is 1 cm past the ischial spines.

■ If the fetus is vertex, the station should be determined by the location of the biparietal diameter (BPD), not the tip-top of the head, which may simply be caput and not the head at all. So, when the BPD is at the level of the ischial spines, the station is 0.

How well did you know this?
1
Not at all
2
3
4
5
Perfectly
266
Q

Define consistency of cervix changes

A

Changes progressively from firm to medium to soft, in preparation for dilation and labor. For some context: Firm consistency is described like the forehead, medium like the tip of the nose, and soft like the check.

How well did you know this?
1
Not at all
2
3
4
5
Perfectly
267
Q

Describe changes position of cervix

A

Describes the location of cervix with respect to the fetal presenting part.
■ Posterior: Difficult to palpate because it is behind the presenting part, and usually high in the pelvis.
■ Midposition.
■ Anterior: Easy to palpate, low in pelvis, pointing forward.
During labor, the cervical position progresses from posterior to anterior.

How well did you know this?
1
Not at all
2
3
4
5
Perfectly
268
Q

Vaginal prostaglandins are inserted for

A

ripening (softening) of cervix

How well did you know this?
1
Not at all
2
3
4
5
Perfectly
269
Q

IV pitocin is used to

A

↑ strength and frequency of contractions

(Pitocin is a synthetic oxytocin. It is used to start or enhance labor.)

How well did you know this?
1
Not at all
2
3
4
5
Perfectly
270
Q

Bishop scoring system used for

A

helps to determine the status of the cervix—favorable or unfavorable—for successful vaginal delivery (If induction of labor is indicated, the status of the cervix must be evaluated to help determine the method of labor induction; A score of ≥6 indicates that the probability of vaginal delivery with induction of labor is similar to that of spontaneous labor)

How well did you know this?
1
Not at all
2
3
4
5
Perfectly
271
Q

A 32-year-old woman is evaluated for preconception counseling. She received two doses of the human papillomavirus (HPV) vaccine before 15 years of age; the tetanus toxoid, reduced diphtheria toxoid, and acellular pertussis (Tdap) vaccine 3 years ago; and the influenza and COVID-19 vaccines during the last influenza season. She has no medical problems and takes no medication. Vital signs are within normal limits and a comprehensive physical examination is within normal limits. Laboratory studies reveal a negative rubella antibody titer and a positive varicella antibody titer. Pregnancy test is negative. Which of the following is the most appropriate next step in management?
-Assure her that no additional vaccines are needed before pregnancy
-Advise her to wait until after pregnancy to receive the rubella vaccine
-Recommend receiving the rubella vaccine immediately after conception
-Administer the rubella vaccine at least one month before she attempts to conceive
-Administer the varicella vaccine now

A

Administer the rubella vaccine at least one month before she attempts to conceive

Live vaccines (e.g., varicella, MMR) are contraindicated during pregnancy. Women who are planning to become pregnant who do not have immunity to rubella or varicella should receive the respective vaccine before conceiving and should wait at least 4 weeks following vaccination before attempting to conceive.

This patient who presents for preconception counseling lacks immunity to rubella. Therefore, she should receive the measles, mumps and rubella (MMR) vaccine at least 4 weeks prior to conception.

The rubella vaccine is critical in preconception care, because intrauterine rubella infection can lead to severe birth defects (e.g., hearing loss, cataracts, cardiac abnormalities, bone lesions, growth restriction, and neurologic abnormalities, including intellectual disability) and miscarriage. The rubella vaccine is a live vaccine, and should therefore be administered at least one month before conception to avoid any risk to the fetus and ensure immunity is established before pregnancy.

Vaccination against varicella is similar to rubella. The varicella vaccine is a live attenuated vaccine used for primary prevention of chickenpox and the vaccine is not recommended during pregnancy. Ensuring immunity before pregnancy is crucial to avoid significant maternal and fetal complications associated with varicella infection.

How well did you know this?
1
Not at all
2
3
4
5
Perfectly
272
Q

A 28-year-old G2P1001 patient at 37 weeks complains of severe headaches and black spots in her vision. Her BP are 165/95 and 163/96 and she has 4+ protein on urine dipstick. Her cervical exam is closed, thick, and high. The fetal heart tones are reassuring and she has no contractions. The US shows a fetus that is appropriate for 37 weeks, with normal amniotic fluid index (AFI), and in cephalic position. What is the next best step?

A

This patient has signs and symptoms of preeclampsia with severe features and should be delivered immediately, especially when term. Induction of labor is a safe option in this care. Patients with preeclampsia can have a seizure at any point before, during, or after labor, so seizure prophylaxis with magnesium sulfate is indicated.

273
Q

Eclampsia defined as

A

Defined as seizure or coma without another cause in a patient with pre-eclampsia
(Eclampsia → hemorrhagic stroke → death)

274
Q

HELLP syndrome: Define

A

manifestation of preeclampsia with severe features

■ Hemolysis.
■ Elevated Liver enzymes.
■ Low Platelets.

(Associated with high morbidity, and immediate delivery is indicated. It may occur with or without HTN.)

275
Q

Timeline for HTN disorders of pregnancy

A

Hypertensive disorders of pregnancy impact patients AFTER 20 weeks’ gestation. Patients with elevated BP prior to 20 weeks are considered to have chronic HTN.

276
Q

What is the treatment of choice for seizure prophylaxis in pregnant patient with preeclampsia?
What is its antidote in case of toxicity on this treatment?

A

Magnesium sulfate (MgSO4)

Calcium gluconate

277
Q

Timeline for diagnosis of chronic HTN relative to pregnancy

A

■ ↑ BP outside of pregnancy
■ ↑ BP prior to 20 weeks’ gestation
■ ↑ BP persisting after 12 weeks post-partum

278
Q

In pregnancy, BP in severe range should be treated within 30–60 minutes using

A

Oral nifedipine or IV labetolol or IV hydralazine are all appropriate agents for lowering of severe range BPs

279
Q

Define superimposed preeclampsia:

A

Preeclampsia in the presence of preexisting chronic HTN. Diagnosed with worsening BPs and new or worsening proteinuria. Patients with chronic HTN are at ↑ risk of developing superimposed preeclampsia.

280
Q

Define Gestational HTN:

A

HTN without proteinuria or other signs/symptoms of preeclampsia

(sustained or transient systolic BP ≥ 140 mm Hg and/or diastolic BP
≥ 90 mm Hg occurs after 20 weeks)

281
Q

Define Preeclampsia:

A

Defined as new onset HTN with either proteinuria or end-organ dysfunction (or both) after 20 weeks

end-organ dysfunction:
■ Thrombocytopenia
■ Serum creatinine >1.1 mg/dL or doubling of serum creatinine.
■ LFTs at least twice the normal concentration.
■ Pulmonary edema.
■ Cerebral or visual symptoms (headache, scotomata).

(May qualify for preeclampsia with severe features depending on combination/levels)

282
Q

The only definitive treatment for preeclampsia is

A

delivery

283
Q

The classic clinical triad of ectopic pregnancy consists of

A

abdominal pain, amenorrhoea and vaginal bleeding

284
Q

Ectopic pregnancy is often clinically mistaken for ——— as presenting symptoms of pain, nausea, and fever are shared between the two.

A

Appendicitis

285
Q

The ——— portion of the fallopian tube is the most common site of an ectopic pregnancy.

A

ampullary

286
Q

——— is a folic acid analog that competitively inhibits dihydrofolate reductase and is used to treat ectopic pregnancy and medical abortion.

A

Methotrexate

287
Q

Risk factors for ectopic pregnancy include

A

prior ectopic pregnancy,
history of infertility,
salpingitis,
pelvic inflammatory disease,
ruptured appendix
prior tubal surgery

288
Q

Define a tubal ring:

A

echogenic, ringlike structure found outside of the uterus, on ultrasonography, that represents an early ectopic pregnancy

289
Q

——— is the standard medication used in the treatment of an unruptured ectopic pregnancy.

A

Methotrexate

290
Q

Define a pseudosac:

A

A collection of fluid within the endometrial cavity created by bleeding from the decidualized endometrium and is often associated with an ectopic pregnancy

291
Q

Kartagener syndrome is a syndrome with ——— that results in infertility and an increased risk of ectopic pregnancy.

A

immotile cilia due to a dynein arm defect (primary ciliary dyskinesia)

292
Q

If transvaginal US does not show an intrauterine pregnancy when the ——— are reached, the pregnancy can be considered ectopic.

A

discriminatory β-human chorionic gonadotropin levels

293
Q

Compared to a normal pregnancy, human chorionic gonadotropin is ——— in ectopic pregnancy.

A

Decreased

294
Q

A 25-year-old woman comes to the office due to vaginal bleeding and abdominal pain for the past 2 days. The patient’s last menstrual cycle was 6 weeks ago. She is sexually active with her male partner of 2 years, and they have been using the withdrawal method as contraception. Medical history is unremarkable other than an episode of pelvic pain 2 years ago that resolved spontaneously without treatment. Temperature is 37.0°C (98.6°F), pulse is 80/min, and blood pressure is 135/85 mmHg. Serum hCG is positive, measuring 1200 mIU/mL. Transvaginal ultrasound shows an empty, normal-appearing uterus and no other abnormalities. Physical examination shows mild lower right adnexal tenderness but is otherwise unremarkable. Which of the following is the underlying cause of this patient’s presentation?
-Obstruction of blood flow to the ovary
-Abnormal implantation
-Premature separation of the placenta
-Spread of pelvic inflammatory disease
-Incomplete early pregnancy loss

A

Ectopic pregnancy should be suspected in any woman of reproductive age who presents with abdominal pain and/or vaginal bleeding. The next step is to obtain urine hCG, followed by a transvaginal ultrasound to confirm intrauterine or extrauterine pregnancy.

This patient is presenting with abdominal pain, vaginal bleeding, an empty uterus on vaginal ultrasound, and positive hCG, raising concerns for ectopic pregnancy. The history of pelvic pain that resolved spontaneously implies pelvic inflammatory disease that caused tubal scarring, which puts her at risk for ectopic pregnancy.

The first step in management in any hemodynamically stable female patient of reproductive age with abdominal pain is to obtain urine/serum hCG. If positive, the next step is to obtain a transvaginal ultrasound to assess for an intrauterine sack. If no intrauterine or ectopic pregnancy have been identified, and hCG levels are below 2000 mIU/mL (discriminatory zone for a gestational sac to be visualized by US), serial quantitative serum hCG levels should be checked every 2 days. hCG levels in normal intrauterine pregnancy should rise ≥35% in 48 hours OR double in 72 hours. Once the levels are above the discrimination zone, transvaginal ultrasound should confirm the diagnosis of ectopic or intrauterine pregnancy. If the patient is hemodynamically unstable at presentation, FAST ultrasound and an emergency surgical consultation are appropriate.

295
Q

Ovarian torsion refers to ———. The classic presentation is ———.

A

rotation of the ovary and its ligament, leading to the impedance of the blood supply

acute onset of pelvic pain, nausea, and vomiting, as well as an adnexal mass

296
Q

Though many patients have a lot of ——— associated with preeclampsia, this is NOT part of the diagnostic criteria.

A

edema

297
Q

Preeclampsia with severe features is defined by the presence of one of the following:

A

Severe BP elevation: systolic ≥160 or diastolic ≥110 at least 4 hour apart (if only BP)

OR

Systolic ≥140 or diastolic ≥90 twice at least 4 hour apart
AND other severe sxs, including
■ Neurologic dysfunction: headache, scotomata, altered mental status.
■ Renal abnormality: serum creatinine >1.1 mg/dL or doubling of serum creatinine.
■ Hepatic abnormality: Epigastric or RUQ pain (hepatocellular ischemia and edema that stretches Glisson’s capsule), ↑ AST, ALT ≥ twice the normal level, or both.
■ Pulmonary edema.
■ Thrombocytopenia

298
Q

Define superimposed preeclampsia:

A

■ Defined by new onset of either proteinuria or end-organ dysfunction after 20 weeks in a patient with chronic HTN.
■ For patients with chronic HTN who have preexisting proteinuria, superimposed preeclampsia is defined by worsening HTN or development of severe features

(Patients with chronic HTN are at ↑ risk of developing superimposed preeclampsia.)

299
Q

Relationship between preeclampsia and eclampsia?

A

Eclampsia is considered a form of severe preeclampsia. More commonly it is NOT preceded by preeclampsia.

300
Q

Effect of Magnesium sulfate in preeclampsia

A

Magnesium sulfate prevents seizures in preeclampsia; does not treat HTN.

301
Q

Magnesium toxicity is associated with sxs of ———. Treat with ———.

A

loss of patellar reflexes, respiratory depression, and cardiac arrest

calcium gluconate IV

302
Q

Complication HTN in pregnancy

A

■ Placental abruption
■ Eclampsia
■ Coagulopathy
■ Renal failure
■ Hepatic subcapsular hematoma
■ Uteroplacental insufficiency and fetal growth restriction

303
Q

HTN may be absent in ——— of patients with HELLP syndrome and severe in ———.

A

20%

50%

304
Q

Objectives in management of severe preeclampsia:

A
  1. Prevent eclampsia which can cause intracranial hemorrhage and damage to other vital organs
  2. Deliver a healthy infant
305
Q

Eclampsia:
■ ——— of seizures are before labor.
■ ——— of seizures are during labor.
■ ——— of seizures are post-delivery (most occur within 1 week of delivery).

A

60%

20%

20%

306
Q

ACE inhibitors used to control HTN in pregnancy?

A

ACE inhibitors are contraindicated because they are teratogenic. Use other classes of antihypertensives to control HTN in pregnancy.

307
Q

Importantly, with HELLP syndrome ——— is indicated.

A

immediate delivery

308
Q

HELLP syndrome may occur with or without ———

A

HTN.

309
Q

A 33-year-old woman, gravida 1, para 0, of 29 weeks’ gestation comes to the emergency room because of a tonic-clonic seizure. She is accompanied by her husband, who says that he has never witnessed her having a seizure before. Her temperature is 37.3°C (99.3°F), pulse is 100/min, respirations are 18/min, and blood pressure is 155/105 mm Hg. Physical examination shows bilateral patellar hyperreflexia. Laboratory analysis shows the following selected values:
Alanine aminotransferase (ALT): normal
Aspartate aminotransferase (AST): normal
Total bilirubin: elevated
Direct bilirubin: elevated
Hemoglobin: normal
Erythrocytes: 5,100,000/mm3
Leukocytes: 9,000/mm3
Platelets: low
Urine protein: elevated
Urine erythrocytes: none
Urine leukocytes: none

Which of the following is the most appropriate diagnosis?
-Hemolysis, elevated liver enzymes, and low platelets syndrome
-Thrombotic thrombocytopenic purpura
-Eclampsia
-Placental abruption
-Pre-eclampsia

A

Eclampsia
Eclampsia is characterized by hypertension, proteinuria and seizures after the 20th week of gestation. Definitive treatment is delivery of the baby after maternal stabilization, but further seizures can be prevented with intravenous magnesium sulfate.

Pre-eclampsia is a disorder of pregnancy defined by hypertension (>140/90) and end-organ dysfunction after the 20th week of gestation to 6 weeks postpartum. End-organ dysfunction can manifest as proteinuria (>0.3 g/24 h urine collection), platelet count <100,000/mm3, or serum transaminase level >2 times normal concentration. Preeclampsia is caused by abnormalities in the placental spiral arteries, which can result in maternal endothelial dysfunction, vasoconstriction or hyperreflexia. One complication of pre-eclampsia is eclampsia, which occurs when a patient with pre-eclampsia experiences new-onset generalized tonic-clonic seizures. Eclampsia-associated seizures places mother at an increased risk for stroke. Eclampsia should be treated with IV magnesium sulfate to prevent seizures, antihypertensives, and expedited delivery if necessary.

310
Q

Antihypertensives in pregnancy: drugs, mechanisms, keys side effects

A

Short-Term Control:
■ Hydralazine: direct vasodilator Side effects: SLE-like syndrome, headache, palpitations

■ Labetalol: nonselective β1 and α1 blocker. Side effects: headache and tremor.

■ Nifedipine: calcium channel blocker. Side effects: edema, dizziness.

Long-Term Control
■ Nifedipine

■ Labetalol (can exacerbate asthma)

311
Q

Risk Factors For HTN Disorders of Pregnancy

A

■ Nulliparity
■ Age<16 or >40 years
■ Family history of preeclampsia in first degree relative
■ Chronic HTN
■ Chronic renal disease
■ Antiphospholipid syndrome
■ Diabetes mellitus
■ Multiple gestation
■ History of preeclampsia in prior pregnancy

312
Q

Reactive NST:

A

15 bpm above the baseline lasting 15 seconds; 2×/20 minutes

313
Q

A 27-year-old woman at 38 weeks of gestation comes to the emergency department with three days of right upper quadrant abdominal pain, nausea, vomiting and fatigue. She had preeclampsia in a prior pregnancy but has an otherwise unremarkable medical history. Her blood pressure is 145/87 and lab studies show anemia with Hgb levels 9.5 g/dL, thrombocytopenia with a platelet count of 55,000/mm3. Liver function tests are abnormal with AST levels at 180u/L, ALT levels at 200u/L, and total bilirubin at 2.0 mg/dL (direct bilirubin is 0.1 mg/dL). A blood smear reveals schistocytes. The patient is admitted to labor and delivery, where fetal testing reveals a heart rate of 95 bpm with minimal baseline variability and late decelerations. Which of the following is the most appropriate next step in management?
-Give ursodeoxycholic acid
-Expectant management
-Caesarian section
-Monitor enzyme levels closely until delivery date
-Platelet transfusion

A

HELLP syndrome is characterized by the presence of hemolysis, elevated liver enzymes, and low platelets. In this situation, an emergency caesarian section is indicated.

This patient is suffering from HELLP syndrome, as indicated by the presence of hemolysis, elevated liver enzymes, and low platelets. The etiology of this disorder is not well-understood, but it is believed to be part of the spectrum of preeclampsia/eclampsia disorders. Therefore, like severe preeclampsia, HELLP syndrome may also constitute an indication for emergency delivery in order to avoid further complications and maternal mortality, particularly if there are also signs of fetal distress. In this case, the fetus is at term (38 weeks) and is presenting with bradycardia and late decelerations. Therefore, it is necessary to proceed with caesarian section for the sake of both mother and fetus.

314
Q

The non-stress test ( NST) evaluates what four components of the FHR tracing:

A

■ Baseline: Normal is 110–160 beats/min.

■ Variability: Beat-to-beat variation of the FHR. Presence of variability reflects an intact and mature brain stem and heart

■ Decelerations:
- Early deceleration: Vagally mediated, caused by head compression usually at cervical dilation of 4–7 cm.
- Variable deceleration: Caused by cord compression.
- Late deceleration: Reflects uteroplacental insufficiency.

■ Acceleration: Presence of accelerations = fetal well-being. Reactive NST = two or more accelerations over 20 minutes.

315
Q

——— decreases plasma bile acid levels is used to relieve pruritus and treat intrahepatic cholestasis of pregnancy

A

Ursodeoxycholic acid

316
Q

Baseline rate for the FHR refers to:

A

the most common heart rate over the last ≥10 minutes

317
Q

The normal baseline for the FHR is

A

110–160 beats per minute (bpm)

318
Q

Periodic changes above and below baseline FHR are termed

A

accelerations (↑ in HR) and
decelerations (↓ in HR)

319
Q

A reassuring NST has

A

two accelerations of at least 15 bpm above the baseline, lasting for at least 15 seconds, in 20 minutes

(It indicates a well-oxygenated fetus with an intact neurological and cardiovascular system)

320
Q

When reading the fetal tracing, always note the following:

A
  1. Baseline
  2. Variability
  3. Presence of accelerations
  4. Presence of decelerations
  5. Contractions
321
Q

NST for fetus <28 weeks’ gestation

A

A fetus <28 weeks’ gestation age is neurologically immature and thus is not expected to have a “reactive” FHR. Thus, an NST is not a good test for fetal well-being in this population.

322
Q

A 29-year-old woman, gravida 2, para 1, at 32 weeks’ gestation comes to the office because of mild abdominal pain in her right upper quadrant for the past three days. She has vomited twice since her pain started. Her temperature is 36.8°C (98.2°F), pulse is 88/min, respirations are 16/min, and blood pressure is 150/95 mm Hg. Physical examination shows facial puffiness as well as right upper quadrant tenderness that radiates into the epigastric area. A urinalysis shows 3+ proteinuria. Which of the following sets of laboratory values are most appropriate to make the diagnosis?
-Elevated fasting glucose level and elevated liver enzymes
-Low platelet count and elevated magnesium level
-Low white blood cell count and elevated liver enzymes
-Low platelet count and elevated liver enzymes
-Elevated blood urea nitrogen and elevated creatinine

A

Low platelet count and elevated liver enzymes
Preeclampsia refers to hypertension with proteinuria and edema in a pregnant woman that arises in the third trimester of pregnancy. It can lead to hemolysis, elevated liver enzymes, and low platelets (HELLP) syndrome.

Preeclampsia is a hypertensive disorder associated with pregnancy that manifests after the 20th week of gestation. It is characterized by a combination of hypertension, proteinuria and nondependent edema in a pregnant woman. The presence of right upper quadrant (RUQ) pain further suggests the development of hemolysis, elevated liver enzymes, and low platelets (HELLP) syndrome, which occurs in about 10-20% of patients with severe preeclampsia. Patients with HELLP syndrome commonly experience nausea, vomiting, and malaise in addition to the features associated with preeclampsia. All of the following criteria must be met for a diagnosis of HELLP syndrome:
Microangiopathic hemolytic anemia with schistocytes on blood smear
Platelets <100,000/μL
Total bilirubin >1.2 mg/dL
Serum aspartate aminotransferase (AST) >twice the upper limit of normal

323
Q

Early Decelerations: significance, when, why, initial tx

A

significance: benign

when: Nadir of decel = Peak of contraction

why: head compression

initial tx: none required

324
Q

Variable Decelerations: significance, when, why, initial tx

A

significance: Variable

when: Variable

why: cord compression

initial tx: aminotransfusion (Infuse normal saline into the uterus through the IUPC to alleviate cord compression)

325
Q

Late Decelerations: significance, when, why, initial tx

A

significance: abnormal

when: gradual decrease below the baseline with onset, nadir, and recovery occurring after uterine contraction onset, peak, and recovery

why: Uteroplacental insufficiency (blood without enough oxygen)

initial tx: O2, lateral decubitus position, Pitocin off, close monitoring

326
Q

The ——— position is best for maximizing cardiac output and uterine blood flow.

A

left lateral recumbent

(In the supine position, the vena cava and aortoiliac vessels may be compressed by the gravid uterus.)

327
Q

STOP when you see decelerations:

A

SVE.
Turn the patient to her left side. Give the patient Oxygen.
Pitocin off.

328
Q

A 30-year-old woman, gravida 2 para 1, comes to the labor and delivery triage at 37 weeks gestation due to mild vaginal bleeding for the past 2 hours. She has not had any pain but has had occasional, non-painful contractions. She did not receive prenatal care but states that she took prenatal vitamins throughout the pregnancy. The patient’s firstborn was delivered via cesarean section. Temperature is 37.0°C (98.6°F), pulse is 100/min, and blood pressure is 115/80 mmHg. On physical examination, the patient appears comfortable. The uterus is soft and nontender. Streaks of blood are seen in the vaginal vault with no active bleeding. The fetus is in a vertex position. The fetal heart rate is 150 bpm. Which of the following is the most likely diagnosis?
-Placental abruption
-False labor
-True labor
-Placenta accreta
-Placenta previa

A

Placenta previa

Placenta previa refers to placental tissue implantation over, or in proximity to, the cervical os. Placenta previa should be suspected in any pregnant woman who presents with painless vaginal bleeding after 20 weeks of gestation. Diagnosis is made via ultrasound, and a cesarean section is typically necessary for delivery.

This patient is presenting with painless vaginal bleeding in the 3rd trimester, favoring the diagnosis of placenta previa.

Placenta previa refers to placental tissue implantation over the internal cervical os, covering all or part of the cervix. The classic presentation is painless late-pregnancy vaginal bleeding. Normally, the placenta implants in the upper uterus, and although it is unclear why it implants in the lower uterus in placenta previa, one hypothesis suggests that areas of suboptimal vascularity in the upper uterine cavity from previous damage (e.g. surgery) promotes implantation in the lower part. Significant risk factors include previous cesarean deliveries, previous uterine surgery, multiple gestations, and previous placenta previa. It is important to note that the presence of pain and tender uterus makes the diagnosis of placenta previa unlikely.

The vast majority of placenta previas are identified on ultrasound examination before 20 weeks gestation and typically resolve spontaneously before delivery. In asymptomatic patients < 37 weeks of gestation, management includes follow up with transvaginal ultrasound. If the previa resolves, routine prenatal care can resume. In unstable patients with severe bleeding or evidence of fetal distress, emergent cesarean surgery is indicated. In patients whose previa does not resolve, cesarean section is the preferred method of delivery to avoid postpartum hemorrhage.

329
Q

Placenta accreta refers to a placenta that ———. Placenta accreta presents as a:

A

attaches to the myometrium without penetrating it

life-threatening hemorrhage that occurs after delivery when manual placental separation is attempted

330
Q

“False labor,” also known as———, refers to

A

Braxton-Hicks contractions

regular or irregular painful contractions without cervical dilation

331
Q

Placental abruption is the ———. Typical presentation includes:

A

premature separation of the placenta from the uterine wall that occurs after 20 weeks gestation

acute onset of abdominal pain, vaginal bleeding (although bleeding may be concealed), uterine contractions, and uterine tenderness

332
Q

Placenta previa is defined as the attachment of placenta to the

A

lower uterine segment over the internal cervical os

333
Q

Placenta previa presents with

A

painless bleeding, most commonly after 20 weeks of gestation

334
Q

———(drug class) are used to help the fetal lung mature in patients with placenta previa

A

Corticosteroids

335
Q

Placenta previa presents with painless bleeding most commonly in the ——— trimester.

A

third

336
Q

——— (type) of placenta previa is when the placenta is within 2 cm of the internal cervical os but does not fully cover it.

A

Marginal

337
Q

A 26-year-old woman comes to the emergency department because of lower abdominal pain and vaginal bleeding for the past 24 hours. Her last menstrual period was 7 weeks ago. She is sexually active with her boyfriend and they use condoms intermittently. Physical examination reveals mild tenderness in the left lower quadrant and left adnexa. The cervix is closed with dried blood visible at the os; there is no active bleeding. A quantitative serum B-HCG is 1,500 IU/L. Transvaginal sonography of the patient’s left adnexa reveals an extra-uterine gestational sac with a diameter of 2 cm. Which of the following is the most appropriate management option for this patient?
Laparoscopic salpingectomy
Open salpingectomy
Misoprostol
Suction and curettage
Intramuscular methotrexate

A

Intramuscular methotrexate
Medical management of ectopic pregnancy using methotrexate is an option for hemodynamically stable women with a small gestational sac and low b-hCG levels.

There are three possible approaches to treatment of ectopic pregnancy: medical, surgical, and expectant management.

Methotrexate is a folic acid antagonist that inhibits cell replication therapy and is used intramuscularly for medical management of ectopic pregnancy. This treatment option requires careful patient selection and is most successful in patients with a b-hCG is <2000 IU/L, and a gestational sac <3.5cm. Medical management is not an option if the patient is not hemodynamically stable. There are several contraindications, including liver and kidney disease, and it is important that the patient has access to and is willing to engage with the required follow up.

Surgical management is indicated for any patient who is medically unstable, is hemorrhaging, has a gestational sac >3.5cm, or a higher b-hCG level (making medical management is more likely to fail). Laparoscopic and open salpingectomy or salpingostomy are the various options for surgical management.

Expectant management is reserved for those patients presenting with symptoms and signs suggestive of ectopic pregnancy but who do not have the expected rise in b-hCG or a visible ectopic mass. These patients undergo regular b-hCG monitoring to ensure the levels are decreasing and are followed until the levels reach zero.

338
Q

A 32-year-old G2P1001 patient at 40 weeks’ gestation presents to labor and
delivery with contractions. She is noted to be 5 cm dilated, 50% effaced, –1 station, and cephalic. The monitor shows contractions every 2 minutes, and the FHR pattern shows a baseline of 150 bpm, minimal variability, and decelerations that nadir after the peak of the contractions. No accelerations are noted. What is the most likely cause of the findings on the FHR? What is the next step in management?

A

Uteroplacental insufficiency is the most likely cause for this patient’s late decelerations. The patient should be turned on her left side to maximize oxygenation to the fetus. IV fluid bolus can be given. If her membranes are ruptured, a fetal scalp electrode and an IUPC (internal monitors) will help with the FHR monitoring.

339
Q

Criteria for fetal tachycardia

A

If an FHR of 160 bpm lasts for ≥10 minutes, then tachycardia is present.

340
Q

Passive immunity for fetus provided by transplacental crossing of ———

A

IgG

(IgG “goes” across the placenta)

341
Q

——— is the antibody present in breast milk.

A

IgA

342
Q

Key complication of varicella zoster infection and tx?

A

Can cause pneumonia

treat with IV acyclovir

343
Q

Varicella vaccine administration during pregnancy?

A

Varicella vaccine is a live vaccine, so cannot be used in pregnancy.

344
Q

Varicella infection can cause severe maternal ——— in pregnancy.

A

pneumonia

345
Q

In pregnancy, post-exposure prophylaxis with ——— within 10 days of exposure is indicated for those who are exposed and susceptible

A

VZIG (varicella-zoster immune globulin)

346
Q

A 25-year-old G1P0 patient at 15 weeks’ gestation reports that she came in contact with a child that had chickenpox 2 days ago. She does not recall ever having chickenpox or the vaccine. What is the next step?

A

She should be tested for the presence of varicella antibodies. Many people are immune to chickenpox, but do not recall ever having it or being vaccinated. If testing indicates that she lacks the antibodies, she should receive the varicella immunoglobulin within 96 hours. If she has the varicella antibodies, nothing further needs to be done

347
Q

Any pregnant patient exposed to influenza should receive prophylaxis with

A

neuraminidase inhibitor (oseltamivir or zanamivir)

348
Q

Influenza vaccine in pregnant patients?

A

Inactivated influenza vaccine should be given to all pregnant patients at any GA.

349
Q

Parvovirus (B19) causes

A

erythema infectiosum or fifth disease

350
Q

Parvovirus (B19) presentation

A

Flulike symptoms are followed by bright red rash on the face—slapped-cheek appearance. Rash may become lacelike, spreading to trunk and extremities.

351
Q

——— in a fetus after parvovirus infection is due to aplastic anemia.

A

Hydrops

352
Q

Fetal hydrops =

A

Collection of fluid in two or more body cavities:
■ Scalp edema
■ Pleural effusion
■ Pericardial effusion
■ Ascites

353
Q

A 35-year-old G4P2012 patient at 26 weeks is diagnosed with anti Kell antibodies with the titer of 1:32. Amniocentesis shows that the fetus is positive for the Kell antigen. In addition to antenatal (i.e., biophysical profile), what other testing is critical for this fetus?

A

The fetus should be monitored with middle cerebral artery (MCA) Dopplers, which indicate the severity of anemia.

354
Q

A 30-year-old G2P1001 patient at 24 weeks’ gestation presents with a bright red rash on both of her cheeks that started yesterday. She reports a 2-day history of fever of 100.4°F (38°C) and lethargy. On physical exam, she is afebrile and has a fine erythematous, lacelike rash on her arms. What is the most likely diagnosis? What is the risk to the fetus?

A

Flulike symptoms, slapped cheek rash, and fine reticular rash (erythema infectiosum) are a classic presentation for parvovirus infection. The fetus is at risk for aplastic anemia, which can cause nonimmune hydrops and fetal death.

355
Q

A 29-year-old G2P1001 patient at 16 weeks presents for prenatal care. Her blood type is A negative and she has a positive antibody screen. What is the next step in management?

A

Identify the antibody. Some can be dangerous for the fetus and some are benign.

(every patient should have her blood type, Rh status, and antibody screen evaluated at the initial prenatal visit)

356
Q

List key antibodies that pose no harm to the fetus vs antibodies that can cause hemolytic disease of the newborn (HDN) and can be fatal?

A

antibodies that pose no harm to the fetus: anti-Lewis

antibodies that can cause hemolytic disease of the newborn (HDN) and can be fatal: anti-D, anti-Kell, anti-Duffy

(Maternal antibodies: Kell Kills, Duffy Dies, Lewis Lives)

357
Q

What Is Rh or D?

A

The surface of the human RBC may or may not have a Rho (Rh) antigen. If a patient with blood type A has a Rho antigen, the blood type is A+. If that person has no Rho antigen, the blood type is A–

358
Q

If the mother is D negative and the father is D positive, baby will be

A

there may be a chance that the baby may be D positive

359
Q

If the mother is D negative and her fetus is D positive:

A

she may become sensitized to the D antigen and develop antibodies against the baby’s RBCs. In her next pregnancy, if the baby is D positive, these antibodies cross the placenta and attack the fetal RBCs, resulting in fetal RBC hemolysis

360
Q

Disease process known as HDN

A

fetal RBC hemolysis results in significant fetal anemia, resulting in fetal heart failure, hydrops, and death

361
Q

What is the most appropriate timeframe for the following prenatal tests/interventions: Rh (D) immune globulin in an Rh- female

A

28 weeks (not for Rh- mom + Rh- dad)

362
Q

What is nuchal cord?

A

Nuchal cord occurs when loops of umbilical cord wrap around the fetal neck.

363
Q

Kleihauer-Betke test determines

A

the number of fetal RBCs in the maternal circulation

364
Q

MOA RhoGAM

A

When a patient is given a dose of anti-D IgG, the antibodies bind to the fetal RBCs that have the D antigen on them and clear them from the maternal circulation. The goal is to prevent the patient’s immune system from recognizing the presence of the D antigen and forming antibodies against it.

365
Q

RhoGAM indication:

A

Give to D-negative patients, who have not formed antibodies against D antigen.

(Not indicated for patients who already have anti-D antibodies and are sensitized.)

(Indicated for patients who might be sensitized to other blood group antigens.)

366
Q

Critical titer for antibody screen in pregnant women:

A

1:16

367
Q

Kleihauer-Betke (KB) test estimates the ———. The dose of ——— is based on the results of the KB test.

A

number of fetal RBCs that are present in the maternal circulation

anti-D IgG

368
Q

Hemolytic Disease of the Newborn: HDN/fetal hydrops occurs when

A

Mother lacks an antigen present on the fetal RBC → fetal RBCs in maternal circulation trigger an immune response → maternal antibodies lyse fetal RBCs → fetal anemia → fetal hyperbilirubinemia + kernicterus + heart failure, edema, ascites, pericardial effusion → death

369
Q

Antibody screen for D-Negative patient should be done at:

If antibody screen negative:

A

the initial prenatal visit and again at 28 weeks

the fetus is to be D positive, and one dose of anti-D IgG immune globulin is given to the mother at 28 weeks to prevent development of maternal antibodies

370
Q

At birth, the infant’s D status is tested. If the infant is D negative, ———. If the infant is D positive, ———.

A

no anti-D IgG is given to the mother

anti-D IgG is given to the mother within 72 hour of delivery (The dose of anti-D IgG is determined using a KB test)

371
Q

If the antibody titer for pregnant women is ——— and/or rising, the likelihood of HDN is high. ——— is recommended.

A

≥1:16

Amniocentesis

372
Q

If during serial US monitoring for fetal anemia (bc prior positive antibody testing), evidence of fetal anemia on MCA Dopplers:

A

Proceed with inutero blood
transfusion (IUT). At this time, assessment of fetal antigen status can be performed if not done previously. If close to term, consider delivery.

373
Q

——— deserve prophylactic Rh (D) immune globulin (even before 28 weeks).

A

Invasive procedures (say amniocentesis)

374
Q

With the use of anti-D immune globulin, there is an ↑ of isoimmunization caused by minor antigens acquired by incompatible blood transfusion. Those antigen that can cause HDN are managed the same way as anti-D isoimmunized mothers. ——— isoimmunization is an exception because: (3)

A

Kell

■ It results in more severe anemia than alloimunization due to other erythrocyte antigens because it also causes fetal marrow suppression in addition to hemolysis.
■ Maternal Kell antibody titers are not predictive of the severity of fetal anemia in the same way that titers are with anti-D sensitization.
■ MCA Dopplers are accurate in predicting severe anemia with Kell isoimmunization.

375
Q

A 29 yo G2P1 F presents for her first prenatal visit at 10 weeks gestation. She is Rh (D) -ve. Her anti-D immune globulin titer is negative. What is the NBSIM of this patient?

A

Leave things as is. Prevent sensitization by administering Rh (D) immune globulin at 28 weeks. Give this earlier if mom has a potential baby blood-mom’s blood mixing procedure like an amniocentesis, miscarriage, etc.

376
Q

A 29 yo G2P1 F presents for her first prenatal visit at 10 weeks gestation. She is Rh (D) -ve. She has a +ve anti-D immune globulin titer. What is the NBSIM of this patient?

A

Get the dad’s blood type. If it’s -ve, the fetus has to be RH -ve. You won’t need to do anything. If the paternal blood type is Rh (D) +ve, there is a good chance the fetus will be Rh (D) +ve. There are some details along the way that are probably “too detailed” for the shelf but a “high anti-RhD titer in mom” should prompt tests for fetal anemia (like a doppler US of the MCA or sampling of fetal blood). If the fetus is anemic, consider an intrauterine blood transfusion and/or delivery.

377
Q

C

A
378
Q

Most common causes of PID

A

Rarely is a single organism responsible for PID (most infections are polymicrobial), but always think of chlamydia and gonorrhea first (these are most common)

379
Q

2 Requirement for a clinical diagnosis of PID:

A
  1. Lower abdominal or pelvic pain
  2. Adnexal, uterine, or cervical motion tenderness on exam
380
Q

CMT is a significant clinical finding that often indicates

A

peritoneal irritation

(CMT, uterine and/or adnexal tenderness are defining characteristics of acute symptomatic PID)

381
Q

Define PID

A

Inflammation of the female upper genital tract (uterus, tubes, ovaries, ligaments) caused by ascending infection from the vagina and cervix

382
Q

PID may lead to

A

tubal scarring and an increased risk of ectopic pregnancy and/or infertility

383
Q

Remember, any reproductive-aged (approximately 15–55 years) female who presents with abdominal or pelvic pain should

A

always get a urine pregnancy test

384
Q

RFs PID (5)

A

■ Age <25 years
■ Multiple sexual partners
■ STI in the partner
■ Unprotected intercourse
■ History of STI or PID

385
Q

Presentation PID

A

■ Lower abdominal or pelvic pain
■ Adnexal, uterine, or CMT
■ Oral temperature >101°F
■ Purulent cervical or vaginal discharge
■ It should also be noted that PID can be asymptomatic or subclinical. In these
cases, PID sequelae (adhesions or hydrosalpinx) are most often identified later during evaluation for infertility or abdominal surgery for other indications.

386
Q

Lab findings in PID

A

■ Gram stain of discharge with gram-negative diplococci (Neisseria gonorrhoeae)
■ Presence of abundant WBCs on microscopy of vaginal secretions
■ Pelvic abscess (i.e., tuboovarian abscess or TOA)
■ Elevated WBC count
■ Culture evidence of N. gonorrhoeae or C. trachomatis.

(Positive lab tests are not necessary for diagnosis. PID is a clinical diagnosis)

387
Q

Laparoscopy findings in PID

A

Not often performed to diagnose PID, but may be helpful in patients who do not respond to antibiotic therapy, or in whom the diagnosis is not clear and other diagnoses need to be excluded (i.e., appendicitis).

■ Reveals pus draining from the fallopian tubes, edema, adhesions, purulent drainage in the cul-de-sac.
■ perihepatic inflammation, known as Fitz–Hugh–Curtis syndrome

388
Q

Fitz–Hugh–Curtis syndrome (perihepatic inflammation)?

A

Ten percent of patients with acute PID will develop perihepatic inflammation, known as Fitz–Hugh–Curtis syndrome. “Violin string” adhesions can be seen at the liver capsule on laparoscopy with this syndrome. Often seen months to years later during laparoscopy for other indications.

389
Q

Association between IUDs and PID

A

Intrauterine devices (IUDs) do NOT increase the risk of PID.

390
Q

Criteria for Hospitalization with PID

A

■ Peritonitis or surgical emergency cannot be excluded.
■ GI sxs, i.e., nausea, vomiting (inability to take oral meds).
■ Abscess (TOA or pelvic).
■ Uncertain diagnosis.
■ Outpatient treatment failure.
■ Immunocompromised.
■ High fever >100.9°F, severe pain.

391
Q

Criteria for hospitalization for PID:

A

GI symptoms
Uncertain diagnosis
Peritonitis
Abscess

392
Q

Inpatient tx for PID

A

■ Cefoxitin/cefotetan + doxycycline.
■ Clindamycin + gentamicin.

393
Q

Outpatient for PID

A

■ Ceftriaxone + doxycycline ± metronidazole.
■ Cefoxitin + probenecid + doxycycline ± metronidazole.
■ Sexual partners should also be treated empirically.

394
Q

Adjunctive care recommendations for PID

A

■ Refrain from intercourse until completed antibiotic therapy.
■ Screen for other STIs.
■ Vaccinate against hepatitis B and HPV if the patient has not been vaccinated previously.

395
Q

Chadwick’s sign

A

Pelvic examination revealing a bluish color of the vaginal wall and cervix - pregnancy sign

396
Q

Key history for ectopic pregnancy

A

history of STI alone or with pelvic inflammatory disease

397
Q

Unruptured ectopic pregnancy classically presents with

A

lower abdominal pain, vaginal bleeding, and abnormal menstrual history

(Physical examination may be completely normal, however classic signs are diffuse abdominal tenderness and unilateral adnexal or cervical motion tenderness)

398
Q

Ruptured ectopic pregnancies are a surgical emergency. On physical exam, ——— suggests intraperitoneal bleeding and ——— correlates with the degree of blood loss

A

abdominal guarding

hypotension

399
Q

A 22-year-old woman presents to the emergency department for evaluation of bilateral lower abdominal pain for 3 days and vaginal discharge. Today she developed nausea and vomiting. She states that she has multiple sexual partners with inconsistent use of condoms. Past medical history is unremarkable. Vital signs are within normal limits. Physical examination reveals lower abdominal tenderness, without rebound or guarding. Labs are ordered and pending. Urine HCG is negative for pregnancy. Which of the following additional findings is most likely to be present on physical examination?
-Thick white discharge on speculum examination
-Cervical motion tenderness
-Vaginal bleeding
-Rovsing sign
-Positive fluid wave

A

PID can be clinically diagnosed if cervical motion, uterine, or adnexal tenderness is noted on physical examination in a patient with a clinical picture that is consistent with PID.

This patient presents with lower abdominal pain and vaginal discharge for 3 days, as well as nausea and vomiting. Given her symptoms, sexual history, and negative pregnancy test, PID is the most likely diagnosis. Examination findings characteristic of PID include lower abdominal tenderness and adnexal, cervical motion, and uterine tenderness. A clinical diagnosis of PID can be made if any of the following is present on pelvic exam: cervical motion tenderness, uterine tenderness, or adnexal tenderness in a patient with a clinical picture that is consistent with PID.

This patient presents with lower abdominal pain and vaginal discharge for 3 days, as well as nausea and vomiting. Given her symptoms, sexual history, and negative pregnancy test, PID is the most likely diagnosis. Examination findings characteristic of PID include lower abdominal tenderness and adnexal, cervical motion, and uterine tenderness. A clinical diagnosis of PID can be made if any of the following is present on pelvic exam: cervical motion tenderness, uterine tenderness, or adnexal tenderness in a patient with a clinical picture that is consistent with PID.

Patients with PID may present with lower abdominal or pelvic pain, vaginal discharge, nausea, vomiting; and they may have dysuria and dyspareunia. Patients may have a history of multiple sexual partners and a lack of contraception use. Typical physical examination findings include lower abdominal tenderness. Rebound and guarding may be seen in more severe disease. In addition, pelvic exams will reveal adnexal, uterine or cervical motion tenderness. A friable cervix with mucopurulent discharge may be seen on pelvic exam.

(Thick white discharge on speculum examination is suggestive of candidal vaginitis, not PID, which is the likely diagnosis in this patient and typically presents with mucopurulent discharge)

400
Q

Epidemiology of simple and complex ovarian cysts in post-menarchal adolescent females?

A

Simple and complex ovarian cysts are relatively common in post-menarchal adolescent females

(Simple cysts result from the failure of a maturing follicle to ovulate and involute)

401
Q

Ovarian cysts presentation

A

While often asymptomatic, ovarian cysts can cause pelvic pain, irregular menses or (when large) reports of pelvic “heaviness,” urinary frequency, and constipation.

402
Q

Complications of ovarian cysts include

A

cyst rupture with varying severity of intraperitoneal bleeding and ovarian torsion

US is the first line of diagnosis

(Clinicians should maintain a high index of suspicion for ovarian torsion when evaluating an adolescent female with acute, severe lower abdominal pain with nausea/vomiting particularly if there is a history of ovarian cysts or an ovarian mass.)

403
Q

All sexually active women younger than 25 years should be tested for

A

gonorrhea and chlamydia every year

(This can be done with urine-based or vaginal swab tests as an alternative to cervical swabs)

404
Q

Key considerations for treating gonnorhea

A

Treat gonorrhea with dual therapy in order to decrease development of antibiotic resistance and treat concurrent chlamydial infections

405
Q

In what media does Neisseria gonorrhea grow?

A

Thayer-Martin in CO2-enriched environment

406
Q

Polymenorrhea definition

A

frequent menstrual bleeding (frequency, 21 days or less) (refers to a menstrual interval of less than 21 days)

407
Q

Menorrhagia definition

A

prolonged or excessive uterine bleeding that occurs at regular intervals (the loss of 80 mL or more of blood that lasts for more than 7 days)

408
Q

A 15-year-old girl is brought to the emergency department with intermittent lower abdominal pain. The patient woke up this morning with intermittent, sharp pain in the left lower quadrant that radiates to the lower back. The pain episodes are associated with nausea and vomiting and spontaneously resolve after a few minutes. She has no nausea or vomiting between episodes and no urinary symptoms, vaginal bleeding, or abnormal vaginal discharge. The patient has no chronic medical conditions and takes no daily medications. Her last menstrual period was 2 weeks ago, and she is not sexually active. Temperature is 37.2 C (99 F), blood pressure is 130/84 mm Hg, pulse is 118/min, and respirations are 20/min. On examination, the patient appears comfortable and has mild tenderness to palpation in the left lower quadrant. Urine pregnancy test is negative. Pelvic ultrasound shows a 5-cm left ovarian mass with normal ovarian arterial and venous blood flow. After the ultrasound, the patient has another episode of severe pain with nausea when walking back to her room from the restroom. On reevaluation, she is in acute distress and has severe, constant tenderness to palpation in the left lower quadrant. Which of the following is the best next step in management of this patient?

A

Diagnostic laparoscopy

Partial ovarian torsion typically presents with intermittent, self-resolving episodes of unilateral pelvic pain due to temporary ovarian vessel occlusion. Patients with intermittent partial torsion can suddenly develop complete torsion, which requires diagnostic laparoscopy.

409
Q

Partial ovarian torsion presentation

A

Patients classically have nausea, vomiting, and intermittent pain that self-resolves (ie, no symptoms between episodes) as spontaneous adnexal untwisting allows blood flow to return. Therefore, Doppler ultrasound may show normal ovarian arterial and venous blood flow.

410
Q

Complete ovarian torsion, typically triggered by ———, presents with ———.
Management is with ———

A

physical activity (eg, walking)

severe, constant, unilateral pelvic pain due to ongoing ovarian ischemia

diagnostic laparoscopy for manual detorsion of the adnexa and removal of any contributory cysts or masses (oophorectomy may be required if the ovary is necrotic)

411
Q

Treatment of N. gonorrhoeae

A

IM dose of ceftriaxone

If chlamydial infections have not been excluded, then additional antibiotics are indicated. Although doxycycline 100 mg BID x 7 days is recommended, for patients who are pregnant or allergic to doxycycline, azithromycin 1 gram orally x1 is also acceptable.

412
Q

Reasons to hospitalize a PID patient include:

A

-Pregnancy
-Previous non-adherence with prescribed medications
-High fever or severe illness
-Intractable vomiting/inability to tolerate oral medications
-Inability to exclude a surgical emergency
-Inadequate response on oral therapy within 72 hours
-Tubo-ovarian abscess

413
Q

ACOG recommends universal screening of all pregnant people for GBS colonization via rectovaginal swab at

A

36 0/7 - 37 6/7 weeks unless GBS bacteriuria was detected during the pregnancy or the patient had a previous newborn was infected with GBS

414
Q

For those who test positive for colonization, intrapartum antibiotic prophylaxis with ———should be given unless ———

A

penicillin or ampicillin

a penicillin allergy is reported (First-generation cephalosporins are the alternative for patients reporting a penicillin allergy that is not anaphylaxis. Patients at high-risk for anaphylaxis should receive clindamycin if the GBS is susceptible to clindamycin. If the GBS is not susceptible to clindamycin, vancomycin is recommended)

415
Q

The goal is that GBS antibiotic prophylaxis will begin ——— prior to delivery for optimal protection of the newborn.

A

at least four hours

416
Q

For patients who are not screened for GBS, antibiotic prophylaxis is recommended for those with certain risk factors such as

A
  • delivery at less than 37 weeks gestation
  • prolonged rupture of membranes greater than 18 hours
    -history of GBS positivity in a previous pregnancy
417
Q

A fetus ——— gestation age is neurologically immature and thus is not expected to have a “reactive” FHR. Thus, an NST is not a good test for fetal well-being in this population.

A

<28 weeks’

418
Q

FHR BTBV The variability ↓ with:

A

■ Fetal acidemia
■ Fetal asphyxia (Hypoxia with metabolic acidosis)
■ Maternal acidemia
■ Drugs [narcotics, magnesium sulfate (MgSO4), barbiturates, etc.]
■ Acquired or congenital neurologic abnormality

419
Q

Absent FHR BTBV =

A

Fetal acidosis, and delivery should be expedited.

420
Q

Fetal tachycardia can be caused by

A

maternal fever or infection and fetal anemia or hypoxia

421
Q

Non-reassuring fetal status with regard to FHR BTBV

A

Minimal (5 or fewer beats per minute), absent, or marked (greater than 20 beats per minute) variability of the fetal heart rate

422
Q

Reassuring Fetal Status markers include

A

-Moderate variability (6 to 20 beats per minute).
-A subjective report of active fetal movement.
-A “reactive” strip showing two heart rate accelerations (of at least 15 seconds with a peak of at least 15 beats per minute above the baseline) in a 20-minute period.

423
Q

Active labor is associated with

A

strong regular contractions every three to five minutes and a cervical dilation of more than 6 cm in the setting of contractions

(The fetal heart tracing does not impact the diagnosis of active labor)

424
Q

First stage of labor (latent and active) defined by

A

Latent phase: regular contractions have started, but the cervix is less than 6 cm dilated

Active phase: begins when 6 cm dilated; ends when fully dilated

425
Q

Second stage of labor defined by

A

begins at full dilation; ends when the baby is delivered

426
Q

Third stage of labor defined by

A

begins with the birth of the baby; ends with delivery of the placenta

427
Q

Patients with CIN 3 require what tx?

A

cervical conization (or LEEP), which is a diagnostic (ie, evaluation for invasive cancer) and therapeutic (ie, excision of dysplastic lesion) procedure. The excised specimen is assessed for depth of invasion and surgical margins (ie, whether all dysplasia was removed).

428
Q

After cervical conization (or LEEP), in patients without invasive cancer, management is based on surgical margins: Patients with positive surgical margins

A

have residual cervical dysplasia that can progress to cancer. Therefore, these patients require additional surgical excision with either repeat conization (if future fertility is desired) or hysterectomy (if no future fertility is desired)

429
Q

After cervical conization (or LEEP), in patients without invasive cancer, management is based on surgical margins: Patients with negative surgical margins

A

have no evidence of residual dysplasia. Because these patients have persistent HPV infection that has caused prior high-grade disease, they are at high risk for recurrent cervical dysplasia and cancer. Therefore, they require more frequent cervical cancer screenings with HPV-based testing (eg, primary HPV testing [without cervical cytology] or co-testing [with cervical cytology]) at 6 months postprocedure. If the result is HPV-negative, patients require annual HPV-based testing for the next 3 years

430
Q

Uterine fibroids are a common cause of ——— in ———, typically causing ———

A

abnormal uterine bleeding

premenopausal women

heavy, prolonged menses

431
Q

Fibroids typically ——— after menopause

A

decrease in size and become asymptomatic

432
Q

PMB is a common presenting finding for ——— (particularly in ———)

A

endometrial hyperplasia or cancer (all patients with PMB require endometrial evaluation)

older women with obesity

433
Q

Endometrial evaluation for PMB:

A

with either transvaginal ultrasound or endometrial biopsy. Endometrial biopsy can supply a histologic diagnosis, but it is an invasive and uncomfortable procedure. Therefore, PMB can also be evaluated initially with transvaginal ultrasound. Patients with an endometrial thickness ≤4 mm on ultrasound have a low likelihood of endometrial cancer and require no additional evaluation. In contrast, patients with an endometrial thickness >4 mm require an endometrial biopsy to assess for endometrial hyperplasia or cancer.

434
Q

Patients with endometrial hyperplasia managed with

A

medically (eg, progestin) or surgically (eg, hysterectomy)

435
Q

Patients with endometrial cancer are typically managed with

A

surgically

436
Q

——— are conditions that should not cause PMB, unless the patient is using MHT.

A

Uterine fibroids and adenomyosis

437
Q

——— should be considered in postmenopausal patients with rapidly enlarging fibroids and bleeding, although the incidence of this is quite small.

A

Leiomyosarcoma

438
Q

Nursing mothers rarely ovulate within the first ——— weeks after delivery. Non-nursing mothers typically ovulate ——— weeks after delivery.

A

10

6-8

439
Q

——— (3 infections) are excreted in breast milk.

A

CMV, HBV, and HIV

440
Q

Breast engorgement commonly occurs

A

3-5 days after delivery, when colostrum is replaced by milk, resulting in high milk production volumes

(exacerbated by a physiologic increase in interstitial edema from decreasing progesterone levels after delivery)

(Engorgement may also occur during times when milk accumulation exceeds drainage (eg, breastfeeding weaning))

441
Q

Presentation breast engorgement:

A

bilateral symmetric breast fullness with diffuse tenderness, warmth, and erythema (no fever)

442
Q

Tx breast engorgement:

A

Cool compresses, acetaminophen, and nonsteroidal anti-inflammatory drugs may be used for symptom control

Patients should experience improvement as breastfeeding or regular pumping is established

443
Q

Presentation of lactational mastitis

A

Often have flulike symptoms (eg, fever, myalgias) and unilateral breast pain with a focal area of erythema and tenderness

444
Q

Galactocele and plugged ducts present as

A

palpable mass

Galactocele- subareolar, mobile, well-circumscribed, non tender mass, no fever

Plugged duct- focal tenderness and firmness and erythema; no fever

445
Q

Breast abscesses presentation

A

Appear like mastitis and an associated area of fluctuance and axillary lymphadenopathy

446
Q

Mammary candidosis presents with

A

unilateral, stabbing breast pain that is out of proportion to examination and have associated scaling of the skin on the nipple of the affected breast

447
Q

Components biophysical profile

A
448
Q

Pregnancies at ≥41 weeks gestation are at increased risk of ———, which can lead to what relative to NST finding

A

stillbirth

In these pregnancies, placental aging leads to decreased placental function, which limits fetal perfusion during contractions and induces intermittent fetal hypoxemia. On nonstress testing, fetal hypoxemia presents as late decelerations, decelerations that gradually nadir after the contraction’s peak

449
Q

Progressive placental dysfunction can lead to ———, which causes CNS suppression and potential fetal demise. To maintain CNS function, the hypoxemic fetus preferentially distributes blood to the brain. Decreased perfusion to the ——— results in ———. Decreased blood supply to ———

A

chronic fetal hypoxemia (ie, uteroplacental insufficiency)

kidney

decreased urine production and oligohydramnios (single deepest pocket <2 cm)

peripheral muscle results in decreased fetal activity (eg, movement, tone) that may be felt by the mother

450
Q

Interstitial cystitis: epidemiology

A
451
Q

Interstitial cystitis: diagnosis

A
452
Q

Interstitial cystitis: presentation

A
453
Q

Interstitial cystitis: treatment

A
454
Q

Interstitial cystitis: definition and presentation

A

(also known as painful bladder syndrome) ia chronic, painful bladder condition of uncertain etiology.

Patients typically have pain that is exacerbated by bladder filling and relieved by voiding. Symptom onset is typically gradual and worsens over a period of months. Other clinical features include urinary frequency and urgency, chronic pelvic pain, and dyspareunia.

455
Q

Interstitial cystitis: associations

A

IC typically presents in women age >40 and is associated with other chronic pain conditions (eg, fibromyalgia, endometriosis, irritable bowel syndrome), sexual dysfunction, and psychiatric illness (eg, depression, anxiety)

456
Q

Diagnosis of interstitial cystitis

A

largely clinical; however, additional laboratory testing—including a urinalysis, postvoid residual, and sexually transmitted infection screening—is performed to exclude other conditions (eg, cystitis, urinary obstruction, malignancy)

457
Q

Management of interstitial cystitis includes

A

bladder training, fluid management, analgesics, and avoidance of any precipitating agents (eg, caffeine, alcohol, artificial sweeteners)

458
Q

Cystocele present with

A

Cystocele (bladder prolapse into the anterior vaginal wall) may cause dyspareunia and urinary symptoms (eg, frequency), changes on pelvic examination with Valsalva (ie, prolapse)

(cystoceles are common in multiparous women)

459
Q

Pelvic organ prolapse occurs when there is ——— and may be classified according to the location of the herniated pelvic organ ie, (described as occur- ring in the anterior vaginal wall (———), posterior vaginal wall (———), uterus/cervix, or the vaginal vault/vaginal cuff in patients who have undergone a hysterectomy (small bowel———). ———occurs when there is herniation of all three compartments, including uterus, through the vaginal introitus.

A

descent of one or more aspects of the vaginal walls or uterus, allowing nearby organs to herniate into the vagina

cystocele

rectocele

enterocele

Uterine procidentia

460
Q

An overactive bladder presents with

A

urinary urgency (an intense need to urinate) that can be accompanied by nocturia and frequency

(There is no associated dyspareunia or pelvic pain relieved with voiding)

461
Q

A 57-year-old G3P3 patient presents with a sensation of pressure and a bulge in her
vagina that is worse at the end of the day. She is healthy and has no medical problems. Her history is significant for three vaginal deliveries proven to 8 lb, and menopause since age 52. What is the next step in the evaluation of this patient?

A

Take a detailed history of symptoms and perform a complete pelvic exam to assess for POP. Examine the patient in both the supine and standing positions to help determine the severity of the prolapse.

462
Q

Stress Incontinence presents as

A

Loss of urine (usually small amount) with↑ intra-abdominal pressure (i.e., with coughing, laughing, exercise, other effort or physical exertion)

463
Q

Stress incontinence often caused by

A

urethral hypermotility and/or intrinsic sphincter deficiency

(Urethral hypermobility may occur when there is insufficient support of the urethra and bladder neck by the pelvic floor musculature and vaginal connective tissue, resulting in the loss of ability of these organs to close completely against the anterior vaginal wall. Increases in intraabdominal pressure results in the urethra failing to close properly, leading to stress incontinence. Intrinsic sphincter deficiency occurs when there is a loss of the normal intrinsic urethral mucosal and muscular tone that usually keeps the urethra closed.)

464
Q

A 52-year-old G4P4 patient reports sudden urgency to void followed by loss of urine before she makes it to the bathroom. The urge is not precipitated by laughing or coughing, and she does not leak constantly throughout the day. What is the most likely underlying cause of her incontinence?

A

This patient is most likely to have urge incontinence that is caused by unopposed detrusor muscle contraction.

465
Q

Presentation of urge incontinence

A

Sudden feeling of urgency followed by involuntary leakage of urine (can be small or large volume loss). Often accompanied with urinary frequency and nocturia.

466
Q

Caused by of urge incontinence

A

unopposed detrusor contraction. Also called “overactive bladder.”

467
Q

Mixed Incontinence: definition

A

Describes patients with a combination of stress and urge incontinence

468
Q

Overflow Incontinence presents with

A

Constant dribbling +/– continuous leakage with inability to completely empty the bladder

469
Q

Overflow Incontinence caused by

A

detrusor underactivity (due to a neuropathy) or urethral/bladder outlet obstruction

470
Q

Incontinence that presents as continuous urinary leakage may be due to ———. This can occur as a result of:

A

a fistula

■ Prior pelvic surgery
■ Obstetric trauma
■ Radiation

471
Q

A urethral diverticulum is an ——— and can cause ———

A

abnormal outpouching of the urethra

a tender anterior vaginal mass, purulent urethral discharge, increased pain with voiding, urethral tenderness, urinary frequency, and dyspareunia

472
Q

Describe the steps and interpretation of the q tip test

A

A cotton swab is placed in the urethra. The change in angle between the Q-tip and the woman’s body is measured upon straining.

Normal upward change is <30 degrees, and a positive test is one with >30-degree change. A positive test indicates urethral hypermobility

473
Q

Cystometry provides measurements of

(Measurements include post residual volume, volumes at which an urge to void occurs, bladder compliance, flow rates, and capacity. Diagnoses: Stress, urge, and overflow incontinence.)

A

the relationship of pressure and volume in the bladder

474
Q

Overflow incontinence treatment

A

Due to obstruction: Relieve obstruction.

Due to detrusor underactivity: Treat possible neurological causes—diabetes mellitus or vit b12 deficiency

475
Q

Treatment urge incontinence

A

Meds, timed voiding, and lifestyle changes:

Antimuscarinic agents (↑bladder capacity,↓ urge by blocking release of acetylcholine during bladder filling). Most common side effects are dry mouth and constipation.

Mirabegron (beta-3-adrenoreceptor agonist)

Timed voiding: Patient is advised to urinate in prescribed hourly intervals before the bladder fills

Avoid stimulants and diuretics (i.e., alcohol, coffee, carbonated beverages)

476
Q

Treatment stress incontinence

A

Kegel exercises strengthen pelvic floor muscles

α-adrenergic agonists

Topical vaginal estrogen

Incontinence pessary

Surgical repair (Midurethral slings are considered first-line surgical treatment)

477
Q

A 23-year-old G3P2002 patient at 25 weeks presents to triage with fever, nausea, and vomiting for 1 day. She reports back pain and lower abdominal pain. She has a fever of 101.2°F (38.4°C), clear lungs, and right costovertebral tenderness. Fetal heart rate is reassuring. The monitor shows contractions every 2 minutes. Cervix is closed/ thick/high. Urine dip shows many bacteria, leukocytes, nitrites, and ketones. What is the most likely diagnosis? What is the next step in management?

A

The clinical presentation is most consistent with pyelonephritis. She should be admitted to the hospital and given IV hydration and IV antibiotics.

478
Q

Major risk of asymptomatic bacteriuria in pregnancy

A

increased risk for acute pyelonephritis
(and subsequent complications, including acute respiratory distress syndrome and preterm labor and delivery)

479
Q

Because of the risks for pyelonephritis in pregnancy, all women require

A

urine culture screening for asymptomatic bacteriuria at the initial prenatal visit

480
Q

The most common associated pathogen for asymptomatic bacteriuria is

A

Escherichia coli

481
Q

Treatment of pregnant women with asymptomatic bacteriuria require

A

antibiotic therapy; first-line antibiotics include fosfomycin and beta-lactam antibiotics (eg, cefpodoxime, amoxicillin-clavulanate).

As a test of cure, a repeat urine culture is performed after antibiotic treatment due to the risk for persistent or recurrent bacteriuria

482
Q

Risk factors for abruptio placentae are

A

conditions that can induce premature separation of the placenta from the uterus, such as maternal abdominal trauma, preeclampsia/HTN, cocaine, prior abruptly placentae

483
Q

Risk factors for cervical insufficiency include

A

conditions that weaken the structural integrity of the cervix, such as prior cervical trauma (eg, cold knife cone, obstetric trauma) and collagen abnormalities (eg, Ehlers-Danlos syndrome)

484
Q

The most common risk factor for fetal macrosomia (ie, birth weight ≥4.5 kg [9 lb 14 oz]) is

A

diabetes mellitus

485
Q

Risk factors for intraamniotic infection include

A

prolonged membrane rupture and protracted labor, which extend the opportunity for normal vaginal flora to ascend into the uterus and cause infection

486
Q

Late-term pregnancy defined as and post-term pregnancy defined

A

≥41 weeks gestation

≥42 weeks gestation

487
Q

Late- and post-term pregnancies are at risk of

A

uteroplacental insufficiency and chronic fetal hypoxemia, which cause CNS suppression and intrauterine fetal demise; To prevent this suppression, blood is preferentially distributed to the brain rather than peripheral tissue. This redistribution can be evidenced on US as oligohydramnios (single deepest pocket <2 cm or amniotic fluid index <5 cm) as amniotic fluid is dependent on renal perfusion and urine production.

(placental function due to age-related placental changes (eg, infarctions, calcifications) that cause increased placental vascular resistance)

488
Q

Complication of late term pregnancies

A

Fetus:
Macrosomia
Dysmaturity syndrome
Oligohydramnios
Demise

Maternal:
Caesarian delivery
Severe obstetric lacerations
Post-partnum hemorrhage

489
Q

Risk factors for late term pregnancies

A

Nullparity

Age > 34

Obesity

Fetal anomalies (anencephaly)

Prior post-term pregnancies

490
Q

Management of post-term pregnancies

A

Frequent fetal monitoring (NST)

Delivery prior to 43 weeks

491
Q

Because the risk of ——— increases with gestational age, patients with late- and post-term pregnancies require frequent fetal monitoring (eg, ———). Patients with findings suggestive of ———, require ———

A

uteroplacental insufficiency

nonstress test, ultrasound for amniotic fluid volume

uteroplacental insufficiency, such as late decelerations or oligohydramnios

delivery to prevent intrauterine fetal demise

492
Q

Risk factors for placenta accreta include

A

prior cesarean delivery (particularly in patients with a placenta previa in the current pregnancy) and prior uterine myomectomy

493
Q

Polyhydramnios (single deepest pocket ≥8 cm or an amniotic fluid index of ≥24 cm) is associated with

A

congenital fetal malformations (eg, esophageal atresia, anencephaly)

maternal diabetes mellitus

494
Q

Risk factors for preeclampsia include

A

extremes of maternal age (eg, <18, >40)
nulliparity
chronic hypertension
diabetes mellitus
renal disease

495
Q

In the brain, preeclampsia increases the risk of

A

ischemic and hemorrhagic stroke

496
Q

Treatment of pre-eclampsia with severe features

A

Antihypertensive therapy (eg, IV labetalol, IV hydralazine)
Treatment of preeclampsia with severe features is with delivery because removal of the placenta is typically curative

497
Q

Management preeclampsia

A

<37 weeks and no severe features: expectant

> = 37 weeks or >=34 weeks with severe features: delivery

Severe range BP: IV labetolol, IV hydralazine, PO nifedipine

Seizure prophylaxis: magnesium sulfate

498
Q

RFs for chlamydia and gonorrhea in women

A

Age <25

High risk sexual behavior

499
Q

Manifestations for chlamydia and gonorrhea in women

A

Asymptomatic (most common)

Cervicitis

Urethritis

Periherpatitis (futz high Curtis syndrome)

500
Q

Dx of for chlamydia and gonorrhea in women

A

Nucleus acid amplification test

501
Q

Tx of for chlamydia and gonorrhea in women

A

Empiric: ceftriaxone and doxycycline (azithromycin in pregnancy)

Confirmed chlamydia: doxycycline (azithromycin in pregnancy)

Confirmed gonorrhea:
Ceftriaxone

502
Q

Complications for chlamydia and gonorrhea in women

A

PID

Ectopic pregnancy

Infertility

Pharyngitis

503
Q

Presentation acute cervicitis caused by Chlamydia trachomatis or Neisseria gonorrhoeae

A

mucopurulent cervical discharge and a friable cervix (ie, bleeding on contact); some patients also have concomitant urethritis (eg, dysuria, positive leukocyte esterase with negative nitrites) due to the proximity of the urethra to the vagina.

(CMT a sign of ascending infection (ie, PID), may be absent with low bacterial count or in early infection)

504
Q

Gonorrhea is often asymptomatic and therefore may go untreated, causing complications of

A

adhesions/inflammation such as ectopic pregnancy, chronic pelvic pain, and infertility

505
Q

A 19-year-old G2P2 patient presents with known exposure to gonorrhea 7 days prior. The patient reports an ↑ in vaginal discharge for the past day, but denies any other symptoms. On physical exam, you notice minimal vaginal discharge. You obtain a nucleic acid amplification test (NAAT) for gonorrhea. What is the next step?

A

Treat the patient empirically. Since the patient notes to a recent exposure to gonorrhea, do not wait for the result to come back. In female patients, asymptomatic infection is common and symptoms may not begin until 7–21 days after exposure.

506
Q

There is a ———% chance of transmission after one exposure to gonorrhea

A

50–90

507
Q

Patients diagnosed with any STI are at an increased risk for ——— and should be tested.

A

HIV

508
Q

For patients with acute cervicitis, ——— testing for C trachomatis and N gonorrhoeae is recommended.

A

nucleic acid amplification (However, empiric treatment prior to the return of results is needed to prevent transmission to sexual partners and to decrease the risk for long-term sequelae (eg, ectopic pregnancy, infertility)).

509
Q

First-line empiric treatment for N gonorrhoeae

A

dual therapy with ceftriaxone plus doxycycline
(Ceftriaxone covers N gonorrhoeae, a gram-negative diplococcus, and doxycycline covers C trachomatis, an atypical organism.)

510
Q

During pregnancy,——— is used in place of ——— (a potential teratogen) for C trachomatis coverage

A

azithromycin

doxycycline

511
Q

Endometrial stripe ——— mm in a patient with PMB should prompt an evaluation for malignancy.

A

> 4–5

512
Q

RFs vaginal cancer

A

Age >60

Tobacco use

HPV infection

In utero DES exposure (Clear cell adenocarcinoma only)

513
Q

Clinical features vaginal cancer

A

Vaginal bleeding

Irregular vaginal lesion

Malodorous vaginal discharge

514
Q

Vaginal squamous cell carcinoma presents with

A

Although many women are diagnosed with vaginal cancer on routine screening, those with large, symptomatic lesions often have vaginal bleeding and malodorous vaginal discharge.

(Additional clinical features that suggest metastatic disease include pelvic pain, urinary symptoms (eg, hematuria), and bulk symptoms (eg, constipation))

515
Q

Vaginal squamous cell carcinoma typically appear as

A

an irregular plaque or ulcer in the upper third of the posterior vagina

516
Q

Cervical cancer and vaginal cancer results from

A

persistent HPV infection with high-risk types 16 and 18

(Chronic tobacco use decreases the immune response and prevents viral clearing)

517
Q

Reiter Syndrome (due to chlamydial infection):

A

Classic triad of conjunctivitis, urethritis, and reactive arthritis: Can’t see, can’t pee, can’t climb a tree.

518
Q

Diethylstilbestrol exposure in utero can cause

A

infertility (due to anatomic defects)

vaginal clear cell adenocarcinoma

519
Q

Patients with Lynch syndrome (hereditary nonpolyposis colorectal cancer, or HNPCC) have a 40–60% lifetime risk of developing

A

endometrial cancer, which is equal to their risk of developing colorectal cancer

520
Q

Any factor that lowers the level or time of exposure to ——— ↓ the risk of endometrial cancer.

A

estrogen

521
Q

RFs for endometrial cancer

A
522
Q

Use ——— rather than ——- for pregnant patients with chlamydia. (As the latter may cause ———)

A

azithromycin

Doxycycline

discoloration of the fetal teeth

523
Q

Clinical features of AFLP

A

Nausea, vomiting

RUQ/epigastric Pain

Fulminant liver failure

524
Q

Management of AFLP

A

Immediate delivery

525
Q

Lab finding in AFLP

A

Elevated bilirubin

Elevated aminotransferases (2-3x normal)

Thrombocytopenia

DIC

Profound hypoglycemia

526
Q

Acute fatty liver of pregnancy usually develops in the _____ trimester, particularly in patients with _____ .

A

third

multiple gestation

527
Q

AFLP presents with

A

hepatic inflammation (eg, epigastric/right upper quadrant pain, leukocytosis, elevated aminotransferases) and subsequent fulminant liver failure (eg, scleral icterus, hyperbilirubinemia, profound hypoglycemia)

(As AFLP progresses, patients can develop multiorgan system failure, including DIC (eg, hemolytic anemia, thrombocytopenia) and acute kidney injury (from hepatorenal syndrome))

528
Q

Centrofacial hyperpigmentation that spares the nasolabial folds in pregnancy

A

Melasma, a common benign condition in pregnancy

(This is likely induced by estrogen and progesterone effects on melanocytes, causing increased melanin accumulation. It typically fades postpartum, although some hyperpigmentation may persist.

529
Q

Define polymorphic eruption of pregnancy (also known as pruritic urticarial papules and plaques of pregnancy [PUPPP])

A

May be triggered by the skin stretching over the enlarging gravid uterus that causes inflammation and pruritus within abdominal striae. It is a benign self-limited condition that resolves postpartum.

530
Q

Increasing upper lip hair in pregnancy?

A

Normal. Due to increased circulating androgens (placental and ovarian), causing thickened terminal hair growth. In addition, scalp hair spends more time in the anagen growth phase, causing decreased hair loss. Hair loss occurs postpartum due to a large proportion of hair in the resting telogen phase (ie, telogen effluvium); however, terminal hairs may be permanent.

531
Q

Spider angiomas in pregnancy?

A

Normal, benign, erythematous lesions that blanch with pressure. They typically resolve postpartum.

532
Q

Define labial adhesions

A

Thin, fused labia minora (Labial adhesions can be partial (involving a portion of the labia) or complete)

533
Q

Labial adhesions most commonly seen in

A

Prepubertal girls due to low estrogen production (Inflammation from poor hygiene, infection (eg, vaginitis), irritation (eg, diaper rash), or trauma (eg, straddle injury) also contributes to the development of adhesions)

534
Q

Sxs of labial adhesions

A

Partial adhesions often asymptomatic (Some individuals may experience vaginal pain or pulling. Adhesions covering the urethral meatus can also cause an abnormal urinary stream and an increased risk for recurrent UTIs due to urine accumulation)

535
Q

Tx labial adhesions

A

Mild, asymptomatic adhesions require no treatment. Topical estrogen is first-line therapy for those with symptoms.

536
Q

Candida vulvovaginitis presents with a

A

Beefy-red rash on the vulva as well as vaginal pruritis and discharge

537
Q

Contact dermatitis presents with

A

a pruritic, erythematous rash

538
Q

Lichen sclerosus is characterized by

A

hypopigmented labial lesions

539
Q

Pinworm infection typically presents in ______ with ______

A

school-age children

perianal itching, particularly at night

540
Q

Urethral prolapse presents in ______ with _______

A

prepubertal girls

inflamed, friable tissue in a donut shape at the urethral meatus

541
Q

A 60-year-old G1P1 patient reports vulvar itching. She has a history of lichen sclerosus that previously responded well to topical high potency steroids. On exam, she has a raised, white lesion on her vulva. What is the best next step in the management of this patient? Is this patient at ↑ risk for malignancy? What microscopic changes contribute to the white appearance?

A

The next step in diagnosis is a punch biopsy. An ↑ risk of vulvar carcinoma is associated with lichen sclerosus (LS), and lichen planus (LP). The white appearance is secondary to lichenification.

542
Q

lichen sclerosus (LS) carries ↑ risk of

A

vulvar squamous cell cancers

543
Q

Lichen simplex chronicus (LSC) is a skin condition caused by ——— ; both a primary cause of ——— and a secondary complication of a ———.

A

chronic itching and scratching

vulvar itching

Any inflammatory vulvar disorder

544
Q

Up to 70% of patients with lichen planus will have ——— involvement

A

oral

545
Q

Disorders of the active phase of labor include: protraction and arrest. Define protraction

A

Cervical change slower than expected, with or without inadequate contractions

546
Q

Disorders of the active phase of labor include: protraction and arrest. Define arrest

A

No cervical change for >=4 hours with adequate contractions

Or

No cervical change for >=6 hours with inadequate contractions

547
Q

Treatment for active phase of labor protraction

A

oxytocin

548
Q

Treatment for active phase of labor arrest

A

Caesarean Delivery

549
Q

The first stage of labor begins with _____ and ends when _____. It consists of a latent phase involving gradual cervical dilation and an active phase with rapid dilation. The transition between the latent and active phase occurs _____.

A

the onset of regular contractions

the cervix is 10 cm dilated

at 6 cm dilation

550
Q

The active phase of labor (6-10 cm cervical dilation) has an expected, predictable rate of cervical dilation of

A

≥1 cm every 2 hours

551
Q

Causes of active phase arrest include

A

uterine (eg, inadequate contractions), fetal (eg, malposition, macrosomia), or pelvic (eg, deformity or fracture)

552
Q

Adequate contractions

A

200-250 MVUs

553
Q

Serotypes L1–L3 of C. trachomatis cause ———, most commonly found in ——— areas.

A

lymphogranuloma venereum

tropical

554
Q

Lymphogranuloma venereum presentation

A

■ Primary lesion: Painless papule on genitals.
■ Secondary stage: Inguinal lymphadenitis with fever, malaise, and loss of appetite.
■ Tertiary stage: Rectovaginal fistulas, rectal strictures.

555
Q

Amnioinfusion is performed using an intrauterine pressure catheter and is indicated for

A

fetal heart rate tracing abnormalities (ie, variable decelerations)

556
Q

Indication for an operative vaginal delivery (eg, vacuum-assisted vaginal delivery)

A

during the second stage of labor (10 cm dilation until fetal delivery) to expedite delivery for category III tracings or maternal exhaustion

557
Q

Oxytocin augments labor by ——— and is used for patients who ———

A

increasing contraction frequency and strength

meet criteria for labor protraction (ie, cervical dilation rate <1 cm/2 hr) but not for those who meet criteria for labor arrest

558
Q

Evaluation for female factor infertility often includes hysterosalpingography (HSG) to

A

identify genital tract anomalies and fallopian tube patency

559
Q

What are bicornuate or separate uterus, and how are they addressed?

A

2 unfused uterine horns, types of congenital uterine anomalies

Some congenital uterine anomalies, such as a septate uterus, can be surgically corrected to recreate a single, larger uterine cavity

560
Q

Complications of bicornuate or separate uterus

A

Spontaneous abortion: Congenital uterine anomalies are commonly associated with absent or abnormal vasculature and diminished uterine blood flow. Therefore, pregnancies experience decreased placental perfusion and inadequate nutrition, which can cause fetal growth restriction or recurrent pregnancy loss.

Preterm labor: Because it has 2 separate, smaller uterine cavities, a bicornuate or septate uterus cannot expand fully in volume to accommodate a full-term pregnancy. When the fetus becomes larger than the cavity, uterine overdistension may trigger preterm labor. Uterine anomalies may also adversely affect the structural integrity of the cervix, increasing the risk for cervical insufficiency and second-trimester pregnancy loss.

561
Q

RFs for pyelonephritis in pregnancy

A

Asymptomatic bacteriuria

Age<20

Diabetes mellitus

562
Q

Most common pathogen with pyelonephritis in pregnancy

A

E. coli

563
Q

Complications of pyelonephritis in pregnancy

A

preterm labor

low birth weight

ARDS

564
Q

Treatment of pyelonephritis in pregnancy

A

IV antibiotics

Supportive care

565
Q

Pregnant women are at increased risk for _____ due to increased urinary stasis, mild immunosuppression, and progesterone-related ureteral dilation.

A

acute pyelonephritis

566
Q

Like most patients, pregnant women with pyelonephritis typically present with

A

nausea, high-grade fever, and flank pain

On examination, CVA tenderness (the angle created between the 12th rib and the spine) or flank pain is common due to retroperitoneal inflammation of the kidney.

Retroperitoneal inflammation can also extend to the abdominopelvic cavity. This can cause obstetric symptoms, such as uterine irritability (eg, nonpainful contractions), that can progress to preterm labor (ie, painful contractions with cervical dilation).

(Maternal fever and tachycardia also lead to fetal tachycardia (ie, HR >160/min))

567
Q

Severe pyelonephritis can also cause

A

maternal sepsis, acute respiratory distress syndrome, and fetal compromise (eg, premature birth, low birth weight)

568
Q

Due to the high risk for severe complications, management of acute pyelonephritis in pregnancy is with

A

hospitalization and empiric intravenous antibiotics (eg, ceftriaxone)

569
Q

Ovarian torsion can present with unilateral abdominal pain, anorexia, and low-grade fever (ie, peritonitis); however, it typically begins as intermittent pain followed by sudden-onset, severe pain consistent with complete vessel occlusion. It is not typically associated with flank tenderness

A

unilateral abdominal pain, anorexia, and low-grade fever (ie, peritonitis)
(typically begins as intermittent pain followed by sudden-onset, severe pain consistent with complete vessel occlusion)

570
Q

Degenerating uterine fibroid in pregnancy can present with

A

abdominal pain, low-grade fever, and uterine tenderness

571
Q

Intraamniotic infection (IAI) is defined as

A

maternal fever (without a clear source of infection) plus either maternal or fetal tachycardia, purulent amniotic fluid, or leukocytosis (IAI is a diagnosis of exclusion
and common cause of abdominal pain in pregnancy)

572
Q

Acute appendicitis in pregnancy can present with

A

fever, nausea, anorexia, and right-sided or upper abdominal pain due to uterine displacement of the appendix cephalad; however, appendicitis typically causes peritoneal signs, including rebound or guarding

573
Q

Women age _____ with abnormal uterine bleeding and classic menopausal symptoms (eg, ____) are likely in menopause and require no additional testing. In contrast, women age _____ with these symptoms require measurement of ______.

A

≥45

night sweats, insomnia, difficulty concentrating

<45

FSH, TSH, and prolactin (eg, FSH (to assess for hypothalamic amenorrhea), TSH (to assess for thyroid disorder), and prolactin (to assess for hyperprolactinemia).

574
Q

A 22-year-old G0 patient presents with a 2-day history of severe, sharp right lower quadrant pain. Her LMP was 13 days ago, and she has no significant medical or surgical history. Her pain is not relieved with ibuprofen. Her vitals show T = 98.2, P = 100, BP 85/60. A pregnancy test is negative. Her hematocrit dropped from 38 to 30%. On exam, her abdomen is soft and tender with rebound and guarding. An ultrasound reveals a normal left ovary and a 4-cm right ovary, with a moderate amount of fluid in the cul-de-sac (pouch of Douglas). What is the most likely diagnosis? What is the best next step in management?

A

The most likely diagnosis is a ruptured corpus luteal cyst. She recently ovulated, since her period was 2 weeks ago. She has vitals and an exam consistent with an acute abdomen. Given her drop in hematocrit, the fluid in the cul-de-sac is likely blood from the ruptured cyst. She needs surgical treatment with a diagnostic laparoscopy to diagnose and treat.

575
Q

During embryogenesis, sexual development in genotypically male fetuses (46,XY) occurs due to _____ production

A

testicular hormone

576
Q

The testes produce testosterone, which promotes development of

A

the male internal genitalia (eg, vas deferens, epididymis)

577
Q

Testosterone is also converted to _____ (via ______), which promotes development of

A

dihydrotestosterone (DHT)

5-alpha reductase

the male external genitalia (eg, penis) and prostate

578
Q

Patients with 5-alpha reductase deficiency are unable to convert _____; therefore, they have

A

testosterone to DHT

undescended testes (eg, bilateral labial masses), haveno male external genitalia development, and appear phenotypically female at birth

At puberty, increased testosterone levels cause virilization of phenotypically female patients, which can manifest as nodulocystic acne and clitoromegaly (ie, clitoris protruding from the clitoral hood). Additional features of virilization include voice deepening and increased muscle mass. The testosterone levels at puberty are at the normal male range; therefore, patients develop additional normal secondary sexual characteristics (eg, axillary and pubic hair). Patients have no breast development because testosterone binds to the breast androgen receptor and inhibits breast tissue proliferation.

579
Q

Diagnosis of 5-alpha reductase deficiency in adolescents and adults is based on an

A

elevated testosterone/DHT ratio

(Management is dependent on age and gender identity)

580
Q

Patients with androgen insensitivity syndrome (AIS) have

A

peripheral testosterone resistance due to a defective androgen receptor

581
Q

In contrast to patients with 5-alpha reductase deficiency, patients with androgen insensitivity syndrome (AIS) have

A

No virilization in adolescence (ie, clitoromegaly), and they lack axillary and pubic hair development.

Patients with AIS also have breast development at puberty due to aromatization of testosterone to estrogen and because breast tissue proliferation is not inhibited by a defective androgen receptor.

582
Q

Ovarian hyperthecosis can present in women with

A

virilization, patients are typically postmenopausal and have bilaterally enlarged ovaries

583
Q

Sertoli-Leydig cell tumors are

A

testosterone-producing ovarian tumors that can result in virilization, patients have a unilateral, solid adnexal mass

584
Q

Pathogenesis of 5 alpha reductive deficiency

A
585
Q

Presentation of 5 alpha reductase deficiency

A
586
Q

To prevent alloimmunization, anti-D immune globulin is administered

A

at 28 weeks gestation and repeated within 72 hours of delivery

587
Q

——— is commonly used to determine the dose of anti-D immune globulin.

A

The Kleihauer-Betke (KB) test: Red blood cells from the maternal circulation are fixed on a slide. The slide is exposed to an acidic solution and adult hemoglobin lyses, leaving “ghost” cells. The dose of anti-D immune globulin is calculated from the percentage of remaining fetal hemoglobin.

588
Q

A ——— block can provide pain relief at the time of a vaginal delivery.

A

pudendal nerve

589
Q

Bartholin gland blockage causes a cyst or abscess. Presentation difference?

A

Most often:
Cysts: Asymptomatic
Abscesses: Painful

590
Q

Vulvar lichen planus is a
chronic inflammatory condition that typically occurs in _____.

A

postmenopausal women

591
Q

The most common type of vulvar lichen planus is the _____ variant, in which chronic inflammation causes _____.

A

erosive

desquamation and erosion of mucosal surfaces, including the vulva, vagina, and oral cavity

592
Q

Clinical features of erosive lichen planus include:

A

Glazed, brightly erythematous vulvar erosions with a border of serpentine-appearing, white striae (ie, Wickham striae) that typically cause vulvar pain, pruritus, and dyspareunia.

Acute vaginal inflammation that causes friable vaginal mucosa and a serosanguinous vaginal discharge; chronic inflammation can eventually result in stenosis of the vaginal introitus.

Lacelike, reticular erosions on the gingivae and palate that cause painful oral ulcers and plaque formations on the tongue.

593
Q

Diagnosis of erosive lichen planus may be made clinically but should be confirmed with a

A

vulvar punch biopsy because the clinical features of lichen planus may overlap with those of vulvar cancer

(Patients with lichen planus require evaluation of all mucosal surfaces because erosions may occur in the absence of other symptoms)

594
Q

First-line treatment is with

A

high-potency topical corticosteroids

595
Q

Vulvovaginal candidiasis can present with

A

Vulvovaginal pruritus and erythema, with a thick, white vaginal discharge; some patients (eg, immunosuppressed) can have accompanying oropharyngeal candidiasis (white oral plaques)

596
Q

Lichen sclerosus can present with

A

Vulvar pruritus and white vulvar plaques with associated erosions (typically have no vaginal involvement)

597
Q

Clinical features of vulvar lichen planus

A
  • Women aged 50-60
  • Vulvar pain or pruitis
  • Dyspareunia
  • Erosive variant (most common): erosive, glazed lesions with white border; vaginal involvement with or without stenosis; associated oral lesions
    -Papulosquamos variant: Small pruritic papules with purple hue
598
Q

Diagnosis of vulvar lichen planus

A

Vulvar biopsy

599
Q

Treatment of vulvar lichen planus

A

High-potency topical corticosteroids

600
Q

The ovarian stroma, which is primarily composed of _____ cells, produces the ______.

A

granulosa

ovarian estrogen supply

601
Q

Granulosa cells convert _____ and secrete ______; therefore, uncontrolled proliferation of these cells, as seen in a granulosa cell tumor, results in ______.

A

testosterone to estradiol (via aromatase)

inhibin (which typically inhibits FSH)

high estradiol and inhibin levels

602
Q

Women with an adult subtype granulosa cell tumor have _____ exposure that can result in ______. Additional clinical features may include _____.

A

chronic, unopposed estrogen

endometrial hyperplasia or cancer (eg, postmenopausal bleeding)

breast tenderness and mass-effect symptoms (eg, abdominal distension, ovarian torsion)

603
Q

Embryonal carcinoma is a type of ovarian germ cell tumor that occurs typically in _____. Embryonal carcinoma secretes _____.

A

young women

alpha-fetoprotein and hCG

604
Q

Mature teratomas (dermoid cysts) are benign germ cell tumors that are more common in _____. Mature teratomas can secrete ____.

A

younger women (age 10-30)

thyroid hormone (eg, struma ovarii)

605
Q

C

A
606
Q

Yolk sac tumors are germ cell tumors that are more common in ____ and secrete _____. Patients typically present with ______.

A

young women

alpha-fetoprotein

acute onset of pelvic pain (because yolk sac tumors grow rapidly)

607
Q

Pathogenesis granulosa cell tumors

A

Sex cord-stromal tumor

Increased estradiol and inhibin

608
Q

Clinical features of granulosa cell tumor

A

Complex ovarian mass

Juvenile subtype: Precocious puberty

Adult subtype: Postmenopausal bleeding, abnormal uterine bleeding, breast tenderness

609
Q

Histopathology of granulosa cell tumor

A

Call-exner bodies (cells in a rosette pattern)

610
Q

Mangement of granulosa cell tumor

A

Endometrial biopsy (endometrial cancer)

Surgery (tumor staging)

611
Q

Vulvar lichen sclerosus is a chronic inflammatory condition that causes ——— in ——— populations.

A

thinning of the vulvar skin

hypoestrogenic (such as premenarchal girls and postmenopausal women)

612
Q

Presentation lichen sclerosus

A

Thin, white vulvar lesions and changes in vulvar architecture (eg, adherence of the labia at the midline). These lesions are chronically inflamed and can result in perianal and vulvar pruritus, at times so severe that it awakens affected individuals from sleep. Excessive scratching can result in excoriations, lichenification (ie, thickened skin), and edema of the labia. Lichenification of the perianal region can result in anal fissures and constipation.

613
Q

Recommended evaluation associated with lichen sclerosus

A

Although biopsy is recommended in adults to exclude an underlying malignancy (eg, vulvar cancer), children have no associated malignancy risk and can be diagnosed clinically.

614
Q

First-line treatment for lichen sclerosus for adults and children:

A

superpotent topical corticosteroids(eg, clobetasol)

615
Q

Streptococcal dermatitis typically occurs as a superinfection in patients with a constant irritant (eg, bathing suit) and can cause perianal pruritis and anal fissures. However, patients have a bright, erythematous perianal rash with sharply demarcated borders rather than thin, white skin.

A
616
Q

Colposcopy:

A

Magnified view of the cervix, vagina, and vulva

617
Q

Epidemiology of lichen sclerosus

A

Prepubertal and perimenopausal or postmenopausal women

618
Q

Clinical features of lichen sclerosus

A
  • Thin white wrinkled skin over labia majora/minors, strophic changes that may extend over perineum and around anus
  • Excoriations, erosions, fissures from severe prutitis
  • Dysuria, dyspareunia, painful defecation
619
Q

Workup of lichen sclerosus

A

Punch biopsy of adult onset lesions to exclude malignancy

620
Q

Treatment of lichen sclerosus

A

Superpotent corticosteroid ointment

621
Q

Most hysterectomies start by ligation and transection of the

A

round ligament

622
Q

Oligohydraminos (AFI <5cm) causes:

A

Uteroplacental insufficiency

Renal anomalies

NSAIDs

Preeclampsia

Abruptio placentae

623
Q

Oligohydraminos (AFI <5cm) complications:

A

Preterm delivery

Cord compression

Meconium aspiration

624
Q

Polyhydramnios (AFI >=24cm) causes:

A

DM

Multiple gestations

Esophageal/duodenal atresia

Ancephaly

Congenital infections

625
Q

Polyhydramnios (AFI >=24cm) complications:

A

Cord prolapse

Preterm labor

PPROM

Fetal malpresentation

626
Q

Fundal height (ie, uterine size) is measured during routine prenatal visits in the second and third trimesters as an indirect assessment of fetal growth and amniotic fluid volume. After——— weeks gestation, fundal height in centimeters should directly correlate to ——— with a small variation (eg, ± ———);

A

20

gestational age in weeks

2-3 cm

627
Q

A presumptive diagnosis of GDM can be made with a very elevated 50-g 1-hour GTT (eg, ≥ ——— mg/dL) or confirmed with a 100-g 3-hour GTT

A

200

628
Q

GDM causes a uterine size–greater-than-dates discrepancy due to resulting

A

fetal macrosomia and polyhydramnios.

629
Q

A uterine size–dates discrepancy requires further evaluation with ——— to measure ———

A

ultrasound

fetal growth and amniotic fluid volume

630
Q

Incorrect pregnancy dating is a common cause of uterine size–dates discrepancies. However, if a patient’s pregnancy is dated by ———, this is the most precise method (ie, within 5-7 days) for establishing gestational age

A

an early first-trimester ultrasound

631
Q

Rh alloimmunization can cause ——— with associated ———

A

hydrops fetalis

polyhydramnios and subsequent uterine enlargement

632
Q

Oral contraceptives (OCs) commonly cause

A

mild elevation in blood pressure and sometimes can lead to overt hypertension

633
Q

The risk of hypertension with OC use increases with

A

the duration of OC use

those who have a family history of hypertension

Those who developed hypertension during a previous pregnancy

634
Q

The CDC recommends against the use of ——— in women with hypertension.

A

combination estrogen-progestin OCs

(In patients whose hypertension is caused by OC use, discontinuation can reduce blood pressure and often corrects the condition. The physician should have a discussion with the patient about the benefit of stopping the OC, the risks of drug continuation, and alternate contraceptive methods.)

635
Q

Weight gain in pregnancy based on BMI

A

<18.5 - 28-40 lbs
18.5-24.9 - 25-35lbs
25-29.9 - 15-25lbs
30>= - 11-20lbs

636
Q

Substances to avoid in pregnancy

A

Undercooked fish, meat, eggs

High mercury fish

Unpasteurized dairy products

High caffeine intake (moderate ok)

(Clean raw fruits and veggies)

637
Q

Multiple sclerosis, an autoimmune inflammatory demyelinating disease of the CNS that typically presents in ———. ——— often precedes the development of focal neurologic findings. There is an increased risk of both the initial presentation and subsequent relapses in the ———

An ——— should be obtained, and would show:

A

young adults, especially women of child-bearing age

Extreme fatigue unrelated to activity levels

early postpartum period.
(Although pregnancy is protective for multiple sclerosis)

MRI of the brain and spinal cord

Typically, multiple, scattered, ovoid lesions are seen in the periventricular regions and the spinal cord. Imaging findings can often confirm dissemination in space (multiple CNS locations) and time (age of lesions), which are required to confirm a diagnosis of MS

638
Q

Mullerian agenesis pathogenesis

A

Mullerian system defect

Abnormal development of uterus, cervix, and upper third of vagina

639
Q

Clinical features of Mullerian agenesis

A

Primary amenorrhea

Normal female external genitalia

Blind vaginal pouch

Absent or rudimentary uterus

Bilateral functional ovaries (normal fsh)

640
Q

Management Mullerian agenesis

A

Evaluate for renal tract abnormalities (renal ultrasonography)

Vaginal dilation (surgical or nonsurgical)

641
Q

Presentation of müllerian agenesis (Mayer-Rokitansky-Küster-Hauser syndrome)

A

primary amenorrhea, normal secondary sexual characteristics, and absent uterus

(Paramesonephric duct development does not affect ovarian development; therefore, patients have normal estrogen levels and normal secondary sexual characteristics (eg, breasts, normal stature))

642
Q

Mullerian agenesis due to

A

the failure of paramesonephric duct development

(paramesonephric (Müller) ducts develop into the uterus, cervix, and upper third of the vagina. Patients with müllerian agenesis do not develop these structures, so amenorrhea is typically the presenting symptom.) (The lower two-thirds of the vagina and the external female genitalia develop from different precursors (ie, urogenital sinus and genital tubercle, respectively); therefore, patients with müllerian agenesis have the lower two-thirds of the vagina (ie, blind vaginal pouch, vaginal dimple) and normal female external genitalia)

643
Q

GnRH deficiency would present with

A

primary amenorrhea and lack of secondary sexual characteristics (low GnRH causes a low estrogen level)

644
Q

Androgen insensitivity syndrome presents with

A

phenotypically female

primary amenorrhea due to an absent uterus

no pubic or axillary hair due to a defective androgen receptor

(complete defect in androgen receptor function)

645
Q

Presentation of 5-alpha reductase deficiency have

A

female external genitalia and male internal genitalia (ie, undescended testes) until puberty, at which time they undergo virilization (eg, muscle l growth, voice deepening) due to increased levels of testosterone

(impaired conversion of testosterone to dihydrotestosterone)

646
Q

Person with developmental failure of the urogenital sinus would have

A

vaginal dimple or no vaginal opening but have a uterus and fallopian tubes (urogenital sinus forms the lower two-thirds of the vagina)

647
Q

Risk factor ectopic pregnancy

A

Previous ectopic pregnancy

PID

Previous pelvic or tubal surgery

648
Q

Clinical features of ectopic pregnancy

A

Abdominal pain, amenorrhea, vaginal bleeding

Hypovolemic shock in ruptured ectopic pregnancy

CMT, adenexal, or abdominal tenderness

Potentially palpable adenexal mass

649
Q

Diagnosis of ectopic pregnancy

A

Positive hcg

Tranvaginal US revealing adenexal mass, empty uterus

650
Q

Management ectopic pregnancy

A

Stable: Methotrexate

Unstable: surgery

651
Q

Ectopic pregnancy present with

A

lower abdominal pain
vaginal spotting
nausea/vomiting

An acute increase in pain may occur with tubal rupture, which can lead to hemoperitoneum with blood pooling in the posterior cul-de-sac, resulting in the urge to defecate (due to increased rectal pressure) and potential hemodynamic instability

652
Q

C

A
653
Q

Risk factors for endometrial hyperplasia

A

Excess estrogen (obesity, PCOS/chronic anovulation, nullparity, early menarche/late menopause, tamoxifen use)

654
Q

Endometrial hyperplasia/cancer symptoms

A

Heavy, prolonged, intrermenstrual bleeding or post-menopausal bleeding

655
Q

Endometrial hyperplasia can be due to

A

chronic unopposed estrogen, which causes uncontrolled endometrial proliferation

656
Q

Endometrial hyperplasia management

A

In patients who may desire future fertility, initial management is with progestin therapy (eg, progestin-releasing intrauterine device), which counteracts estrogen’s effects by inhibiting endometrial proliferation and promoting differentiation. Follow-up is with repeat endometrial biopsy (eg, every 3 months). Patients with stable or improved disease continue progestin therapy, whereas those with progression to cancer typically require hysterectomy

657
Q

Evaluation of endometrial hyperplasia/ cancer

A
658
Q

Müllerian agenesis (ie, Mayer-Rokitansky-Küster-Hauser syndrome), or abnormal paramesonephric (müllerian) duct development presents with

A

normal secondary sexual characteristics (eg, breasts, external genitalia), but the uterus and cervix and upper 1/3 vagina are absent

The ovaries and external genitalia develop independently of the müllerian duct system; therefore, patients have normal FSH levels (ie, normal ovarian function), normal external genitalia, and the lower two-thirds of the vagina (ie, blind vaginal pouch)

659
Q

Patients with müllerian agenesis should undergo evaluation of the

A

renal tract (ie, ureters, kidney) with renal ultrasonography

(Because of their common embryologic source and synchronous development in the first trimester, internal genital anomalies and renal abnormalities are often concurrent)

660
Q

——— is performed in patients with Turner syndrome to evaluate for ———. Patient has a normal karyotype and FSH level (ie, normal ovarian function), making Turner syndrome unlikely.

A

Echocardiography

cardiac malformations (eg, bicuspid aortic valve, aortic coarctation)

661
Q

——— levels are measured to diagnose 5-alpha reductase deficiency and androgen insensitivity syndrome (AIS).

A

Dihydrotestosterone and testosterone

662
Q

Presentation 5-alpha reductase deficiency

A

genotypic males (46,XY) appear phenotypically female until puberty; elevated testosterone levels at puberty then cause virilization (eg, clitoromegaly) and lack of breast development

663
Q

Presentation AIS

A

genotypic males appear phenotypically female through puberty and have breast development. Those with AIS have no or minimal acne or pubic hair development due to an abnormal androgen receptor.

664
Q

——— levels are measured in patients with primary amenorrhea who have a uterus and a low or normal FSH

A

TSH and prolactin

(MRI of the head is indicated in patients with a low or normal FSH, high prolactin, or visual field defects to evaluate hypothalamic and pituitary causes (eg, sellar mass).)

665
Q

Gestational Thrombocytopenia defined as

A

Mild isolated (100,000-150,000)

No symptoms

Diagnosis of exclusion

666
Q

Preeclampsia with severe features/HELLP defined

A

Moderate-severe thrombocytopenia (<100,000)

Hypertension with or without headache and scotomata

With or without elevated creatinine, elevated AST/ALT

667
Q

Define gestational thrombocytopenia

A

benign condition that causes an isolated, mild thrombocytopenia (ie, platelets 100,000-150,000/mm3)

Patients are asymptomatic (ie, no bruising, bleeding, or anemia) or have normal physiologic symptoms of pregnancy, such as fatigue, shortness of breath, and symmetric leg swelling

668
Q

Gestational thrombocytopenia management

A

includes reassurance and observation

669
Q

Fetal dysmaturity syndrome occurs in ——— pregnancies due to ———. Neonates present as

A

post-term (≥42 weeks gestation)

age-related placental changes and resultant uteroplacental insufficiency

small for gestational age, have a thin body with loose skin, and have meconium-stained amniotic fluid

670
Q

Types of twins and associated signs on US

A

Dichorionic, diamniotic - lambda sign (thick intertwin membrane)

Monochorionic, diamniotic - t or delta sign (thinner intertwin membrane)

671
Q

Maternal complications with twins

A

Preeclampsia
GDM
IDA
Hyperemesis Gravidarum

672
Q

Monochorionic, diamniotic twins only are at risk for

A

Twin twin transfusion syndrome (A serious complication of monochorionic twin gestation in which blood/ intravascular volume is shunted from one twin to another across the shared placenta.)

673
Q

Fetal complications twin getation

A

Pre-term labor
Congenital anomalies
IUGR
Malpresentation (breech)

674
Q

Monochorionic, monoamniotic twins only are at risk for

A

Cord entanglement
Conjoined twins

675
Q

The most common complication of twin pregnancies is

A

preterm delivery (ie, <37 weeks gestation)

676
Q

Fetal macrosomia, an estimated fetal weight ≥4,500 g, typically occurs in patients with

A

gestational diabetes mellitus

excessive pregnancy weight gain

some syndromes (eg, Beckwith-Wiedemann)

677
Q

Risk factors for placenta accreta include

A

prior uterine surgery (eg, cesarean delivery, myomectomy), particularly in patients with an associated placenta previa

678
Q

Uterine inversion, an obstetric emergency, occurs when the uterine fundus collapses and turns inside out during delivery of the placenta. Risk factors include

A

fetal macrosomia
uterine anomalies
a retained placenta
placenta accreta